SlideShare ist ein Scribd-Unternehmen logo
1 von 107
Downloaden Sie, um offline zu lesen
MCQS AND ASSAY
COVER
The Arab Board for Medical Specializations
Scientific Council for Family And community Medicine
Board Certification
In community Medicine
(PART I)
2007, 2008, (FROM 2014, TO 2021).
Dr.AbdulQawi Almohamadi
abdulqawiqaid.wordpress.com
Contents
COVER__________________________________________________________________ 1
Contents ________________________________________________________________ 2
MCQs 2007______________________________________________________________ 3
MCQs 2008_____________________________________________________________ 14
MCQs 2014_____________________________________________________________ 25
MCQs 2015_____________________________________________________________ 35
Long Questions 2015 _____________________________________________________ 44
MCQs 2016_____________________________________________________________ 45
Long Questions 2016 _____________________________________________________ 53
MCQs 2017_____________________________________________________________ 54
Long Questions 2017 _____________________________________________________ 65
MCQS 2018 ____________________________________________________________ 66
Long Questions 2018 _____________________________________________________ 77
MCQs April 2019 ________________________________________________________ 78
Long Questions April 2019_________________________________________________ 87
MCQs October 2019 ______________________________________________________ 88
Long Questions October 2019 ______________________________________________ 97
MCQs January 2021______________________________________________________ 98
Long Questions January 2021______________________________________________107
Answers Exam January 2021 ______________________________________________107
[2007 arab board for community medicine exam paper one (70%)- mcqs.]
Dr.AbdulQawi Almohamadi
Page 3 of 107
MCQs 2007
1. Epidemiology can be best defined as the study of
A. The etiology of disease in humans.
B. The distribution and determinants of health-related
states or events.
C. The patterns of organization and financing of health
care systems.
D. The frequency of causes of death.
E. The prevention and control of infectious diseases.
2. A slender 50-year-old with fair skin is being seen for a routine
pap smear. She is postmenopausal and in good health with no
known medical problems. She leads a sedentary life mostly
indoors. Which of the following preventive measures is most
appropriate for her?
A. Electrocardiogram.
B. Discussion of aspirin therapy.
C. Discussion of estrogen replacement therapy.
D. Instructions to use skin protection from UV light.
E. None of the above.
3. For a man aged 40 to 50 years, with no known medical
problems, which of the following screening tests should
routinely be performed?
A. Urinalysis.
B. Sigmoidoscopy.
C. Cholesterol.
D. Prostate specific antigen.
E. None of the above.
4. Which of the following is a leading potential cause of death
for a healthy child aged 2 years, for which a preventive
measure is available and which should be addressed with the
parents?
A. Lead toxicity
B. Motor vehicle accident
C. Tuberculosis
D. Pneumonia
E. Influenza
Q5-6:
A pharmaceutical company has developed a new treatment A for a
rare disorder that is associated with a high risk of death. It
conducted a randomized trial which included 200 subjects; half
were assigned the active treatment, and the other half placebo.
Thirty of the subjects receiving treatment died during the course of
follow-up; forty of those receiving placebo did.
5. The relative risk of mortality for drug A compared to placebo
is:
A. 30/100
B. 40/100
C. 30/40
D. 30/200
E. 40-30/ 200
6. The statistical test for the effect of drug A has a P-value (2-
Sided) = 0.027. Which of the following statements are correct
interpretations about this statistical result?
A. The probability that the null hypothesis is true is
2.7%.
B. If in fact drug A does not affect mortality, the
probability of observing a positive result is 2.7%.
C. If in fact drug A does affect mortality, the
probability of observing a positive result is 2.7%.
D. If in fact drug A does affect mortality, the
probability of observing a negative result is 2.7%.
E. None of the above
Q7-8:
A Case-control study was performed to determine the association
between cigarette smoking and acute myocardial infarction (MI).
Cases were patients with MI admitted to a single hospital, and
controls were patients hospitalized at the same hospital without a
diagnosis of MI. Trained interviewers then asked all subjects about
their prior smoking history. Assume that smoking does increase the
risk of MI.
7. What characteristic is NOT appropriate for the "control"
group?
A. Controls should be disease-free.
B. Controls should come from the same population
from which cases were selected.
C. Controls should have the same exposure as cases.
D. Restrictions applied to cases should also apply to
controls
E. Controls should receive full information about the
aims of the study
8. The most specific type of bias potentially found in Case-
Control studies is the...
A. Measurement bias
B. Random bias
C. Recall bias
D. Study cost
E. Loss to follow-up
Results
Drugs
Placebo
Total
Died
30
40
70
Living
70
60
130
Total
100
100
200
[2007 arab board for community medicine exam paper one (70%)- mcqs.]
Dr.AbdulQawi Almohamadi
Page 4 of 107
9. The preferred method for handling dropouts or non-
compliance with treatment in a randomized trial is to...
A. Analyze data according to the treatment actually
received by the patients, not to how they were
randomized.
B. Analyze data as in answer (a), but control for
baseline differences between groups in prognostic
factors.
C. Analyze data as though the patients randomized to a
given treatment actually received that treatment,
even though they may not have received that
treatment.
D. Analyze outcomes of patients who actually
complete the course as intended.
E. Analyze outcomes following the treatment received
or those based on treatments to which patients were
randomized, both are good approaches as long as
blinding technique were correctly used.
10. A flight carried 98 passengers, 47 of whom reported suffering
from gastrointestinal illness within four days of the flight. All
the following statements regarding the attack rate are true,
EXCEPT:
A. The attack rate is the incidence rate of
gastrointestinal illness among the flight passengers.
B. The attack rate is an appropriate incidence rate for a
specific group of individuals, under limited
circumstances.
C. 0.48
D. 0.12 per day.
E. The attack rate may underestimate the real incidence
of disease in this group if some mild cases are
missed.
11. The functions of Public Health surveillance include all the
following EXCEPT:
A. Data collection.
B. Data analysis.
C. Interpretation of reported data.
D. Dissemination of results.
E. Disease control action in response to outbreak
notification
12. The safest method of malaria prevention is:
A. Chemoprophylaxis.
B. Environmental control.
C. Combination of drugs and insecticides.
D. Biological control.
E. Genetic methods.
13. Diseases that may affect the fetus include all, EXCEPT:
A. Toxoplasmosis
B. Rubella
C. Tuberculosis
D. Cytomegalovirus
E. HSV
14. Which of the following is the most effective method for
family planning?
A. Oral contraceptive pills.
B. Withdrawal.
C. Safe period counts.
D. Condom use.
E. Traditional methods.
15. Breast feeding can improve all the following, EXCEPT:
A. Uterine involution.
B. Bonding.
C. Infant nutrition.
D. Infant immune system.
E. Prevention of HIV infection.
16. Of the following environmental hazards, the only one which
causes a non-threshold exposure is ...
A. Ionizing radiation.
B. Noise.
C. Heat.
D. Lead contamination.
E. Carbon monoxide poisoning.
17. The diagnostic power of a test is reflected by its
A. Sensitivity.
B. Specificity.
C. Predictive value.
D. Attributable risk.
E. Validity.
18. Which one is the most Cost-Beneficial technology in
improving public health in developing nations?
A. Safe water supply.
B. Oral rehydration solutions.
C. Hospitals in all communities with at least 1,000
population.
D. One doctor for 500 population.
E. Control of industrial pollution.
19. Folic acid supplementation is important in the prevention of
A. Byssinosis.
B. Intrauterine growth retardation.
C. Pellagra.
D. Neural tube defects.
E. Preeclampsia.
20. All of the following cancers have been associated with
occupational exposure, EXCEPT:
A. Bladder cancer.
B. Lung cancer.
C. Hematopoietic cancers.
D. Breast cancer.
E. Liver cancer.
[2007 arab board for community medicine exam paper one (70%)- mcqs.]
Dr.AbdulQawi Almohamadi
Page 5 of 107
21. How are numerical summaries of clinical measurements
affected by a frequency distribution that is skewed to the
right?
A. The mean and the median are equal.
B. The mean is larger than the median.
C. The median is larger than the mean.
D. The mode is larger than the median.
E. The mean and the median are larger than the mode.
22. The vector of Leishmaniasis is...
A. Culex fatigans.
B. Anopheles mosquito.
C. Phlebotomus sand-fly.
D. Aedes Egypti.
E. Culex mosquito.
23. Risk factors of gout include all the following, EXCEPT:
A. Hyperuricemia.
B. Alcohol consumption.
C. Lead exposure.
D. Female gender.
E. Use of diuretics.
24. A person working in hot environment and taking a lot of
water but no salt will develop a condition known as:
A. Heat stroke.
B. Heat syncope.
C. Heat cramps.
D. Heat exhaustion.
E. Heat hyperpyrexia.
25. Farmers are at risk of all the following, EXCEPT:
A. Malignant pustule.
B. Hydaditosis.
C. Toxic effects of pesticides.
D. Pulmonary anthrax.
E. Chronic skin changes.
26. Health care workers are at risk for developing which of the
following occupational diseases?
A. Silicosis.
B. Bilharziasis.
C. Byssinosis.
D. Bagatosis.
E. Hepatitis B infection.
27. The marginal cost in the context of health care refers to:
A. The cost of delivering one extra unit of services.
B. The cost of training and recruiting staff.
C. The cost of buildings and equipment.
D. The cost of all daily activities.
E. The cost on the consumers of care.
28. Basic resources for providing optimal health care include all
the following EXCEPT:
A. Doctors, nurses and other health personnel.
B. Health centers, hospitals and clinics.
C. Financial budgets.
D. Time factor.
E. Medical schools and health training institutions.
29. The process of day-to-day follow-up activities to ensure that
health programs are proceeding as planned is:
A. Evaluation.
B. Surveillance.
C. Monitoring.
D. Implementation.
E. Prioritization
30. The best indicator for the assessment of the nutritional status
of under-fives children is ...
A. Weight for age.
B. Height for age.
C. Weight for height.
D. Head circumference.
E. Skinfold thickness
31. Which one is the best indicator for the status of iron storage in
the body?
A. Hemoglobin concentration.
B. Total iron binding capacity.
C. Serum iron.
D. Serum ferritin.
E. Mean corpuscular hemoglobin concentration
32. Learning can be affected by the following conditions,
EXCEPT:
A. Gender.
B. Intelligence.
C. Age.
D. Motivation.
E. Mental health.
33. Which of the following factors tends to increase patient's
compliance?
A. The presence of a psychiatric disorder.
B. The need for long-term treatment.
C. Significant behavioral change required.
D. Medicines with few side effects.
E. Treatment with lower costs.
[2007 arab board for community medicine exam paper one (70%)- mcqs.]
Dr.AbdulQawi Almohamadi
Page 6 of 107
Q 34-35:
An investigator wished to determine whether families with a
child with attention deficit disorder (ADD) have other family
members who develop depression more than do families
without children with ADD. Group I was made up of families
with a child identified as having ADD. through a community
psychiatry clinic. Group II was made up of families also seen
at the community psychiatry clinic for diagnoses other than
ADD. The groups were followed over one year and the
incidence of depressive episodes in the two groups was almost
similar.
34. What type of study was this?
A. Prospective cohort.
B. Case-control.
C. Clinical trial.
D. Historical cohort.
E. Exposure study.
35. The measure of association in this case...
A. Requires obtaining also the prevalence rates of
ADD in the population.
B. Can be measured even if one does not know how
large is the population from which participating
families were selected.
C. May be affected by the recall bias.
D. Is the odds-ratio.
E. Indicates that ADD is one family member is a
protective rather than a risk factor for depression.
36. The sensitivity rate of a test is defined as...
A. True negative rate
B. True positive rate
C. True positives + True negatives
D. Hundred minus false positive rates
E. Accuracy rate
37. Cross-Sectional Studies ...
A. Measure the incidence rate of a disease.
B. Measure the prevalence rate of disease.
C. Are useful for investigating rare conditions.
D. Identify the risk of occurrence of disease associated
with specific characteristics.
E. Are useful in investigating disease etiology.
38. Which ONE of the following diseases has the highest case-
fatality rate?
A. Rabies.
B. AIDS
C. Leprosy
D. Hemorrhagic fevers
E. Tuberculosis
39. Aluminum sulfate is commonly utilized in...
A. Milk purification
B. Decreasing air pollution
C. Prevention of radiation hazard
D. Sewage disposal
E. Water purification
40. Solid waste disposal in short-term refugee camps is best done
by...
A. Land filling of shallow trenches.
B. Composting.
C. Throwing on the road side.
D. Incinerating.
E. All of the above.
41. Which one of the following pesticides can cause pulmonary
fibrosis?
A. Organophosphates.
B. Parathion.
C. Halogenated hydrocarbons.
D. Paraquat.
E. Mercury compounds.
42. An important indicator of environmental air pollution is
A. Chlorine.
B. Fluoride.
C. Sulphur dioxide.
D. Carbon monoxide.
E. Hydrogen.
43. The best method for treating water from a very turbid source
is
A. Alum coagulation
B. Rapid sand filtration
C. Activated charcoal
D. Ozonation
E. Aeration
44. In a hospital-based case-control study of the association
between coffee consumption and the occurrence of pancreatic
cancer, a group of patients hospitalized after diagnosis of
pancreatic cancer was compared to a control group
hospitalized for other reasons. The patients hospitalized for
pancreatic cancer (cases) were found to consume significantly
more coffee than the controls. All of the following statements
represent possible explanations for the observed positive
association between coffee consumption and pancreatic
cancer, EXCEPT:
A. Heavy coffee drinkers may also be heavy smokers,
so smoking rather than coffee consumption is the
relevant causal factor.
B. Excessive coffee consumption causes pancreatic
cancer.
C. The hospitalized controls consumed less coffee, on
the average, than individuals in the general
population with no history of pancreatic cancer
resulting in a spurious association between coffee
consumption and pancreatic cancer.
D. The cases restricted their coffee intake after being
diagnosed with cancer.
E. The preferred measure of association in this case is
the odds-ratio.
[2007 arab board for community medicine exam paper one (70%)- mcqs.]
Dr.AbdulQawi Almohamadi
Page 7 of 107
45. All the following statements about observational studies are
true, EXCEPT:
A. Events are observed as they occur in nature, with no
active intervention on the part of the investigator.
B. The comparison groups may differ with respect to
factors related to the response variable.
C. Subjects may be followed forward in time from
exposure to occurrence of outcome, backward in
time from outcome to exposure, or evaluated at a
single point in time to concurrently assess outcome
and exposure.
D. They are especially useful in situations where it is
impossible, impractical, or unethical to manipulate
exposure to a suspected risk factor.
E. None of the above.
46. A pharmaceutical company wishes to evaluate the
effectiveness of an antihypertensive agent. It will be
submitted for regulatory approval only if it can be shown to
lower average systolic blood pressure by at least 10 mmHg
compared to a standard drug. The investigators calculate that
if they administer the new drug and the standard drug to two
groups of 100 subjects each, the study will be 90% certain of
detecting the relevant 10 mmHg difference, if it actually
exists. The power of the proposed study is
A. Greater than the power of a study that included 50
subjects in each treatment group.
B. Is defined as the probability of detecting the 10
mmHg difference in systolic blood pressure between
the comparison groups if no true difference exists.
C. Is equal to 0.90.
D. A & C
E. B & C
47. A Prospective Cohort Study ...
A. Employs subjects known at the onset to have the
disease in question.
B. Employs subjects known at the onset to be disease-
free.
C. Employs subjects whose exposure to a suspected
risk factor is comparable to that of the control group
at the onset of the study.
D. Is preferred in the study of rare diseases.
E. Is more ethical than a case-control study.
48. Vital statistics in most countries include data on all the
following, EXCEPT:
A. Death.
B. Birth.
C. Marriage.
D. Divorce.
E. Disability
49. The International Classification of Diseases (ICD) is
A. A relatively recent concept.
B. Needed to compare disease patterns between
countries.
C. Developed specifically for each of the WHO
regional offices.
D. Established at the Center for Disease Control in
Atlanta (USA).
E. Not available for ill-defined signs and symptoms
often encountered in primary health care practice.
50. The sequential temporality means that
A. That there are more than one cause acting in a
specific order to cause disease.
B. The relation between cause and effect is for limited
time.
C. There is an association between cause and effect
only in specific seasons.
D. The cause has occurred after the effect.
E. None of the above.
51. Biological plausibility means that...
A. The same findings may be observed by different
persons in different places.
B. The association between cause and effect is
biologically possible.
C. Greater levels of exposure lead to greater effects.
D. The effect of similar biological factors should be
considered.
E. None of the above.
52. Which of the following statements is correct regarding
population - based health management?
A. Epidemiology can make a significant contribution to
generating population-based health management
data.
B. This type of health management focuses on
assessing the health status of targeted populations
C. This management assesses the financial resources to
cover the health needs of a population.
D. A & B.
E. A & B & C
53. The attributable risk is defined as the...
A. Incidence of a disease in the exposed group.
B. Incidence of a disease in the non-exposed group.
C. Incidence in the total population.
D. Incidence in the exposed group divided by incidence
in the non- exposed group.
E. Excess risk due to the presence of a risk factor in a
given population.
[2007 arab board for community medicine exam paper one (70%)- mcqs.]
Dr.AbdulQawi Almohamadi
Page 8 of 107
54. Which value represents the strongest significant relative risk?
A. 1.3 (0.8 - 2.1)
B. 0.5 (0.4 - 0.6)
C. 2.4 (1.2 - 3.6)
D. 2.2 (1.2 - 3.6)
E. 2.6 (0.7 - 4.5)
55. What is/are the most specific problem(s) associated with
survey interviews, compared to postal surveys?
A. Higher costs.
B. Lower response rates.
C. Lower validity.
D. More important potential selection bias.
E. All of the above
56. In a case of streptococcal sore throat with at least 4 symptoms,
what is the probability of obtaining a positive throat culture?
A. 2.5%
B. 6-7%
C. 14.1-16.9%
D. 20-55%
E. None of the above
57. All of the following disorders are believed to be solely the
result of genetic factors EXCEPT:
A. Epilepsy.
B. Hemophilia.
C. Fibrocystic disease of the pancreas.
D. Multiple polyposis of the colon.
E. Xeroderma pigmentosum.
58. Treatment of a person with AIDS using AZT is what level of
prevention?
A. Primary prevention.
B. Secondary prevention.
C. Tertiary prevention.
D. Primordial prevention.
E. None of the above.
59. Virulence is defined as...
A. The propensity of Neisseria meningitides to produce
severe and fatal disease.
B. The capacity of the HIV virus to survive in defined
environments outside the human body.
C. The capacity of the influenza virus to enter, survive
and multiply inside the host.
D. The capacity of toxigenic E.coli to produce the
functional, morphologic and pathologic changes that
cause symptomatic disease.
E. The capacity of Salmonella species to resist
common antibiotics.
60. Pathogenicity is defined as...
A. The propensity of Neisseria meningitides to produce
severe and fatal disease.
B. The capacity of the HIV virus to survive in defined
environments outside the human body.
C. The capacity of the influenza virus to enter, survive
and multiply inside the host.
D. The capacity of toxigenic E.coli to produce the
functional, morphologic and pathologic changes that
cause symptomatic disease.
E. The capacity of Salmonella species to resist
common antibiotics.
61. Fertility refers to
A. The capacity to bear children.
B. The probability of conceiving in a given month.
C. The actual birth of living offspring.
D. The probability of conceiving in a given year per
1,000 women.
E. The ratio of live births to total pregnancies.
62. All the following techniques of early detection of
asymptomatic cancer are considered to be justified, EXCEPT:
A. Mammography.
B. Breast self-examination in younger women.
C. Papanicolaou (Pap) smear.
D. Fecal occult blood testing.
E. Chest radiography to detect lung cancer.
63. Town B has a higher crude mortality rate as a result of cancer
in men than in Town A. However, the age-standardized
mortality rate from cancer is higher in Town A than in town
B. What conclusions can be drawn from these data?
A. There were more deaths from lung cancer in Town
A than Town B.
B. There were more deaths from lung cancer in Town
B than in Town A.
C. Town A has a younger age distribution than Town
B.
D. Town B has a younger age distribution than Town
A.
E. None of the above conclusions may be made with
certainty with the given data.
64. The most frequent cancer site in Middle-East women is
A. Lung.
B. Breast.
C. Colorectal.
D. Uterus.
E. Ovary.
65. The oligo-element believed to have the greatest influence in
inhibiting the formation of free radicals which can cause
cancer is
A. Iron.
B. Selenium.
C. Copper.
D. Zinc.
E. Mercury.
[2007 arab board for community medicine exam paper one (70%)- mcqs.]
Dr.AbdulQawi Almohamadi
Page 9 of 107
66. Which of the following statements about child abuse is
correct?
A. Child abusers are most often unrelated to the child.
B. Most child abusers have psychotic or antisocial
personalities.
C. More than 95% of all serious head injuries of
children under the age of one are due to physical
abuse.
D. Child abuse is rare in most Arab countries and
should therefore not be reported to relevant
authorities.
E. None of the above.
67. The weight of high-school children who engage in
competitive sports is compared to that of students who do not.
The best way of testing whether differences are statistically
significant between the two groups is to use:
A. Chi-square test.
B. Correlation coefficient
C. Unbiased distribution
D. Student's t-test
E. Multivariate analysis
68. On a hot summer day in a large urban center, the emergency
room department reports an increase in admission for asthma
in children and young adults but not among patients suffering
from chronic bronchitis. The most likely air pollutant
responsible for exacerbation of asthma is:
A. Co
B. Ozone
C. N02
D. Particulate matters
E. Lead
69. The most important factor for heat-related illness is:
A. Obesity
B. Age over 65
C. Female gender
D. History of prior heat strokes
E. Low socio-economic status
70. Which statement regarding acute flaccid paralysis (AFP) is
not correct?
A. Countries which do not report AFP may not have a
good surveillance system for polio
B. The approximate AFP incidence rate is known for
any given population
C. The reporting of AFP is not mandatory
D. The main differential diagnosis for AFP is polio
E. AFP cases that are not caused by polio are most
often cases of Guillain-Barre syndrome
71. Diseases whose spread is linked to utilization of ventilation
systems in big building is:
A. Lyme's disease
B. Ebola fever
C. Asthma
D. Rubella
E. Legionellosis
72. The target population for early detection of chronic conditions
by periodic health examinations or mass screening programs,
consists of:
A. Asymptomatic, apparently well people
B. Cases diagnosed at the primary health care level
C. Attendants of out-patient facilities
D. Hospitalized in-patients
E. Persons who have signs and symptoms of the
disease
73. The epidemiological transition means that:
A. Chronic diseases are now a thing of the past
B. Nutritional problems are not a major public health
issue any more
C. This generation is expected to survive longer than
its grand-parents
D. Infant mortality rates have been decreasing steadily
E. Infectious diseases are better prevented through
immunization than through personal hygiene
74. If rapidly progressive cancers are missed by a screening test,
which type of bias will occur?
A. Lead-time bias
B. Length bias
C. Selection bias
D. Surveillance bias
E. Information bias
75. "The following figure shows that the height of a son is highly
correlated (r=l.02) with the height of his father. It indicates a
positive linear relationship between the two heights." Which
of the following statements is correct?
A. The association is not linear
B. The correlation is not positive
C. The correlation coefficient (r) should not be used in
this case
D. The value of the correlation coefficient (r ) is not
correct
E. The height of the son is not correlated with the
height of the father
[2007 arab board for community medicine exam paper one (70%)- mcqs.]
Dr.AbdulQawi Almohamadi
Page 10 of 107
76. Electromagnetic fields generated by high-voltage electric
power lines have been incriminated in causing
A. Lymphoma
B. Breast cancers
C. Diarrhea
D. Emphysema
E. Childhood leukemia
77. The fact that the combined effects of several exposures is
greater than the sum of the individual effects is known as:
A. Latency
B. Synergism
C. Threshold effect
D. Dose-response effect
E. Pedestalization
78. Which form of payment encourages physicians to require
more procedures and services than may be needed?
A. Fees paid by a Health Maintenance Organization
capitation system
B. A Public Health System in which doctors receive a
monthly basic salary
C. Fee-for-service without compensation to the patient
D. Fee-for-service with third-party repayment to the
patient
E. None of the above
79. Global warming may be associated with the emergence or re
emerging of all EXCEPT:
A. Dengue
B. Yellow fever
C. Rift Valley fever
D. Malaria
E. Bovine spongiform encephalopathy
80. Knowing that the sample mean is 23, the standard deviation is
0.5 and the sample size is 100. The 95% confidence interval
of the mean would approximately be:
A. [21-25]
B. [22-24]
C. [21-24]
D. [22.5-23.5]
E. [22.95-23.05]
81. Travelers to endemic malaria area should:
A. Get a prophylactic vaccine one week before
departure and a booster within three weeks of
returning
B. Consult a doctor for drug dosages to be used while
on that trip
C. Rely on mosquito repellent and bed screens to
prevent bites
D. Avoid having sex with someone living in that area
E. Remain under medical surveillance for a year if
infection with Plasmodium falciparum is suspected
82. In a study of heart surgery, one issue was the reduction effect
of a drug on the pulse rate of patients during surgery. The
available subjects were divided at random into two groups.
(group1) received the drug, the other (group 2) a placebo.
What would be the null and the alternative hypothesis
statements?
A. Ho: Mean1=Mean 2, and H1: Mean 1>Mean 2.
B. Ho: Mean1=Mean 2, and H1: Mean 1<Mean 2.
C. Ho: Mean1=Mean 2, and H1: Mean 1≠Mean 2.
D. Ho: Mean1<Mean 2, and H1: Mean 1=Mean 2.
E. Ho: Mean1>Mean 2, and H1: Mean 1=Mean 2.
83. In the control and prevention of meningococcal meningitis:
A. The carrier state is common in epidemics
B. Man and monkey are reservoirs of infection
C. Isolation of patients is of significance in disease
control
D. The infectious agent is Neisseria meningitides
E. No specific immunity follows clinical infection
84. The width of a confidence internal will decrease when which
of the following changes occur?
A. The desired level of confidence increases
B. The subject-to-subject variation in values of the
response variable increases
C. The sample size increases
D. The precision of the estimate of the population mean
decreases
E. The distance between the sample mean and the
population mean increases
85. All of the following infectious diseases are described as
zoonosis, EXCEPT:
A. Typhoid fever
B. Leptospirosis
C. Brucellosis
D. Rabies
E. Salmonellosis
[2007 arab board for community medicine exam paper one (70%)- mcqs.]
Dr.AbdulQawi Almohamadi
Page 11 of 107
86. Which of the following is the most common site of hospital-
acquired/ infections
A. Surgical wound
B. Respiratory tract
C. Skin
D. Urinary tract
E. Gastrointestinal tract
87. All of the following statements about tuberculosis are true,
EXCEPT:
A. The risk of developing tuberculosis is greater within
the first year following infection
B. Most cases of tuberculosis occur as a result primary
infection
C. Routine screening of general populations is no
longer recommended
D. INH chemoprophylaxis may be given to selected
high risk patients over 35 years of age
E. Most cases of tuberculosis can be successfully
treated with a 6-month drug regimen
88. The incidence of cholelithiasis is increased in all the
following, EXCEPT:
A. Persons with diabetes
B. Persons with chronic hemolytic anemia
C. Persons with hypercholesterolemia
D. Persons who are obese
E. Women
89. Infants of diabetic mother are not at risk of
A. Polycythemia
B. Hypomagnesaemia
C. Neonatal jaundice
D. Hypoglycemia
E. Traumatic delivery
90. Exposure to dimethyl mercury (organic mercury) found in
medical laboratories may cause all the following EXCEPT:
A. Cerebellar degeneration
B. Paresthesias
C. Gingivitis
D. Coma
E. Visual deficits
91. If the output of a program meets the expected outcome, then
the program is described as:
A. Adequate
B. Productive
C. Efficient
D. Effective
E. Relevant
92. Break point of chlorination means:
A. The start the chlorination process
B. Free residual chlorine starts appearing.
C. The end of chlorination process
D. Partial saturation of water with chlorine.
E. All of the above
93. Diagnostic criteria for the metabolic syndrome according to
the Adult Treatment Panel (ATPIII) include all the following
EXCEPT:
A. Abdominal obesity
B. Dyslipidemia
C. High blood pressure
D. High fasting glucose
E. Microalbuminuria
94. The best food source of iron is:
A. Dates
B. Whole wheat bread
C. Egg yolk
D. Raisins
E. Milk
95. Health equity is best measured by:
A. Infant Mortality Rates
B. Child growth
C. Survival rates
D. Medical rates
E. Primary health care (PHC) facilities
96. High parity may be associated with the following EXCEPT:
A. Hypertensive disorders of pregnancy
B. Cancer of the cervix
C. Breast cancer
D. A high risk of infant mortality
E. Gestational diabetes mellitus
97. Heat stability is lowest for:
A. Diphtheria vaccine
B. Hepatitis B vaccine
C. Dried measles vaccine
D. Dried BCG
E. Sabin vaccine for poliomyelitis
98. The mental illness is most likely to occur in young adults (15-
24) is:
A. Autism
B. Schizophrenia
C. Affective disorders
D. lnvolutional melancholia
E. Agitated depression
99. Which statement concerning oral rehydration solution (ORS)
is correct?
A. Contains salt, sugar and trace of vitamins and
minerals
B. Salt is included to improve the function of the small
intestine mucosa and increase water and sodium
absorption
C. Will help in preventing vomiting of the child
D. Mothers should not stop breast-feeding when giving
the ORS
E. Mothers should not give ORS to their children in all
types of diarrhea
[2007 arab board for community medicine exam paper one (70%)- mcqs.]
Dr.AbdulQawi Almohamadi
Page 12 of 107
100. In maternal care:
A. The safe motherhood initiative is related to the work
by UNISCO to prevent avoidable causes of deaths
among mothers.
B. The four pillars of the safe motherhood initiative
include antenatal care, female nutrition, hospital
delivery and family planning.
C. Risk status of pregnancy is determined from the
history taking during the first visit to the antenatal
D. Age at pregnancy and parity of the mother are
example of non avoidable risk factors during
antenatal
E. Pregnant mothers with previous still births and
abortions are considered among moderate to low
risk group.
101. In school health service, the following are true, EXCEPT:
A. School age groups represent an interval of
development that needs medical supervision.
B. The pattern of health problems in the community
will determine the shape of the school health
service.
C. It is not recommended to have a doctor in each
school as part of the school health service.
D. Families might be affected by health education
given to their children in schools.
E. Teachers have a minor role in the planning and
implementation of the school health service.
102. Regarding the status of newborns, infants and children in the
world, all the following statements are true, EXCEPT:
A. Preterm labor and low birth weight are two major
causes of neonatal and perinatal mortality in both
developing and developed world
B. Birth trauma and infection can be counted as the
major causes of death in the developing world
C. Respiratory distress syndrome is a health problem in
newborn that contributes a high percentage in their
mortality and morbidity
D. Tetanus neonatorum has become a minor cause of
death in the developing world
E. Infant mortality in Saudi Arabia can be reduced
more by the prevention of low birth weight, labor
complication and neonatal infections .
103. Natural passive immunity is obtained through...
A. Immunization
B. Acquired infections
C. Administered antibodies
D. Collective immunity
E. Trans placental antibodies
104. Which infectious agent is associated with high incidence of
liver cancer?
A. Hepatitis A Virus (HAV)
B. Human Papilloma Virus (HPV)
C. Helicobacter pylori
D. Hepatitis B Virus (HBV)
E. Malaria species
105. All the following features of polio may contribute to its
eradication, EXCEPT:
A. There exists a safe and inexpensive vaccine against
polio
B. Most cases of polio infection are asymptomatic
C. Polio may be attacked during a season of low
incidence
D. There is public demand for the eradication of this
disease
E. Natural or induced immunity is life-long
106. Vaccination may fail because of which factor?
A. Some vaccines cannot resist cold weather
B. Vaccines are used in populations different from
those in which they were developed
C. Drugs necessary to activate the vaccine were not
taken as prescribed
D. A vaccine was licensed for use in human
populations without proper controlled trials
E. Vaccinated individuals are too old to develop
antibodies to the vaccine
107. Acute respiratory infections in children
A. Are preventable with measles and whooping cough
vaccination.
B. Are measured with accurate statistics based on
advanced surveillance systems worldwide.
C. May cause death as a consequence of pneumonia.
D. May be transmitted from domestic animals acting as
reservoirs, and such animals should not be allowed
near children.
E. May be equally severe in developing than in
developed countries
108. Control of enteric diseases includes all EXCEPT:
A. Washing hands after touching a patient
B. Using white gowns
C. Isolating the utensils of the patients
D. Restricting visits
E. Using masks when caring for the patient
109. An endemic disease had a steady frequency over time in a
community. Recently, however, a new treatment has been
developed that prevents death but does not produce recovery
from the disease. The following will occur:
A. Incidence of the disease will increase
B. Incidence of the disease will decrease
C. Prevalence of the disease will increase
D. Prevalence of the disease will decrease
E. Both incidence and prevalence of the disease will
decrease
[2007 arab board for community medicine exam paper one (70%)- mcqs.]
Dr.AbdulQawi Almohamadi
Page 13 of 107
110. The "meningitis belt" refers to:
A. A special belt worn by African pilgrims during Hajj.
B. Abdominal pain accompanying meningitis infection.
C. A rush which appears in the mediastinum following
vaccination against meningitis infection.
D. Geographical area in central Africa where
meningitis is endemic.
E. The geographical area in Africa where meningitis
has been eradicated.
111. Cigarette smoking may cause all the following, EXCEPT:
A. Colo-rectal cancer
B. Emphysema
C. Stroke
D. Larynx cancer
E. Bladder cancer
112. The occurrence of an illness at a rate above the expected
number is called:
A. Epidemic
B. Endemic
C. Sporadic
D. Pandemic
E. Zoonotic
113. The "healthy worker effect" causes which kind of bias?
A. Selection
B. Confounding
C. Recall
D. Social
E. Measurement
114. The Health Belief Model (HBM) explains
A. The religious beliefs related to health.
B. The cognitive variables affecting the desire to
engage in health prevention.
C. What people think about causes of serious diseases
such as cancer.
D. Differences in the trust that people put in the
healthcare providers.
E. The association between demographic variables and
health beliefs.
115. What is the maximum quarantine incubation period to allow a
person coming from an infected area to enter a country?
A. One incubation period
B. Double the incubation period
C. Thrice the incubation period
D. One month
E. Two months
116. A man of 63 had been treated for some years for malignant
hypertension. While seriously ill with congestive heart failure,
he developed acute appendicitis. Appendectomy was carried
out successfully, but the heart condition deteriorated further
and he died 2 weeks later. The underlying cause of death was:
A. Malignant hypertension
B. Congestive heart failure
C. Cardiac hypertrophy
D. Acute appendicitis
E. Appendectomy
117. The target population for early detection of chronic conditions
either by periodic health examinations or mass screening
programs consists of:
A. Asymptomatic, apparently well people.
B. Individuals with symptoms and/ or signs of the
disease.
C. Cases diagnosed at the primary health care level
D. Hospitalized in-patients.
E. Users of emergency rooms
118. All the following statements regarding smoking are true,
EXCEPT:
A. As role-models, physicians should abstain from
smoking in public
B. Passive smoking is a serious health hazard for those
sharing living quarters with a smoker
C. It is not acceptable that the majority of male medical
students would be smokers
D. Proper enforcement of rules and regulations should
be considered in any smoking control program
E. Nicotine addiction is a desirable way of coping with
stress
119. Regarding the Gulf War Syndrome:
A. It has become a major public health problem in
Kuwait after Liberation.
B. It is a genetically determined syndrome.
C. It is a clearly defined bacteriological disease.
D. It is the label given by the UN to the malnutrition
and high mortality rates experienced by Iraqi infants
and children after the imposition of sanctions.
E. It is a collection of ill-defined health problems
experienced by soldiers from the coalition who had
received a variety of vaccinations prior to serving in
the Middle-East during the Kuwait Liberation War
120. Dermatitis, diarrhea and dementia are signs associated with
deficiency in vitamin...
A. A
B. B1 (Thiamine)
C. C
D. D
E. B3 (Niacin)
END OF THE EXAM
MCQs 2008 Arab Board For Community Medicine Exam Paper 1 (70%)
Dr.AbdulQawi Almohamadi
Page 14 of 107
MCQs 2008
1. For a man aged 40 to 50 years with no known medical
problems, which of the following screening tests should
routinely be performed?
A. Urine analysis
B. Sigmoidoscopy
C. Cholesterol
D. Prostate specific antigen
E. Electroencephalogram
2. Electromagnetic fields caused by high voltage electrical
lines have been incriminated in causing...
A. Lymphoma
B. Breast cancer
C. Diarrhea
D. Emphysema
E. Childhood leukemia
3. The medical records of 73 people with hepatitis B were
reviewed to determine if there is an association between
their previous vaccination status and their chances of
contracting hepatitis B. This type of study is
A. A case-control
B. A cohort
C. Cross-sectional
D. Correlational
E. Experimental
4. The crude birth rate, the most fundamental fertility
measure, uses
A. All births in the numerator and total population
in the denominator.
B. All births in the numerator and women 15-44
years of age in the denominator.
C. The sum of all the age-specific fertility rates by
single years of age.
D. Birth to women in a specific age range in the
numerator.
E. Total fertility rate minus crude death rate.
5. In an epidemic investigation of an outbreak of unknown
origin, which activity should be performed first?
A. Confirmation of the diagnosis for cases
B. Verification that an epidemic really exists
C. Development and testing of an explanatory
hypothesis
D. Proposal of measures for control of the health
problem
E. Search for additional cases of the suspect
disease
6. When a screening for HIV is performed, the ELISA test
is repeated if positive. If the second test is positive, a
Western blot is performed for confirmation. This serial
interpretation has which of the following effects?
A. Increases both the sensitivity and specificity
B. Increases the predictive positive value and
slightly decreases the sensitivity
C. Increases the predictive positive value and
slightly increases the sensitivity
D. Increases both the specificity and the predictive
negative value
E. Increases the sensitivity only
7. The two most important values usually necessary as a
description of the distribution of a series of quantitative
observations are...
A. Standard deviation and mean
B. Median and variance
C. Mode and range
D. Range and mean
E. Size of sample and standard deviation
8. The proportion of death cases from a specific disease
divided by all deaths is called...
A. An incidence rates
B. A point prevalence
C. A case fatality
D. A proportional mortality rates
E. A disease-specific mortality rate
9. Which of the following risk factors is associated with the
greatest number of cancer deaths?
A. Diet
B. Alcohol
C. Smoking
D. Obesity
E. Environmental exposure
10. A 21-year-old college student presented to the student
health center with complaints of cough and fever for a
few days. An erythematous maculopapular rash and
Koplick's mucosal spots are found on physical
examination. Which of the following statements is true
concerning this illness?
A. This illness is more common and more severe
in children compared with infants or adults
B. In the typical form, the rash is not a common
symptom
C. Conjunctivitis, excessive lacrimation and
photophobia are common symptoms
D. Prompt administration of immune globulins
soon after exposure does not alter the course of
the illness
E. Antibody protection after the infection lasts for
only 2 to 3 years
MCQs 2008 Arab Board For Community Medicine Exam Paper 1 (70%)
Dr.AbdulQawi Almohamadi
Page 15 of 107
Q11-12. :
A 72-year-old man has a 1-month history of angina
pectoris occurring at rest, lasting up to 20 minutes.
Episodes have been increasing in frequency and intensity
over the past few weeks. He has smoked 2 packs of
cigarettes per day for 50 years and has been under
treatment for hypertension for the last 15 years.
11. Which type of angina best describes this patient's pain?
A. Chronic stable angina
B. Variant angina (Prinzmetal's)
C. Silent ischemia
D. Unstable angina
E. Traditional angina
12. The initial step in the management of patients should be:
A. Admission to the hospital for further work-up
B. Observation for several hours in the emergency
room until pain resolves
C. Obtain ECG and follow patient on outpatient
basis if ECG is normal
D. Place patient on aspirin 325 mg per day and
instruct him to return to an office follow-up
visits in two
E. Reassure patient and send him home without
further follow- up
13. Which of the following is NOT true about the prevention
and treatment of Haemophilis influenzae infection?
A. Chemoprophylaxis with isoniazid for
household contacts under the age of 6 has been
recommended in cases of H.influenzae
meningitis
B. For normal healthy infants, H.influenzae
immunization begins at 2 months of age
C. A three-stage vaccination is available for
primary immunization and a booster
D. Treatment for life threatening illness from the
H.influenza may include both chloramphenicol
and ampicillin.
E. Protection with the vaccine lasts 5 to 10 years
or longer.
14. The following are primary preventive measures of
diabetes EXCEPT:
A. Genetic counseling
B. Encouraging exercise
C. Controlling weight
D. Giving appropriate doses of insulin
E. Encouraging complete immunization against
infection
15. Precautions in the administration of oral polio vaccine
include avoidance of vaccination in the presence of
which of the following conditions?
A. An Immunodeficient household contact
B. Prematurity
C. Diarrhea
D. Breast feeding
E. Low-grade fever
16. A pregnant 23-year-old women comes into the primary
care center with a diagnosis of pelvic inflammatory
disease. She is at LEAST risk for developing which of
the following
A. Premature rupture of membranes
B. Spontaneous abortion
C. Chorioamnionitis
D. Prematurity
E. Low birth weight infant
17. A 36-year-old obese man with documented diastolic
hypertension presents to your office. Which of the
following is LEAST likely to help improve his diastolic
hypertension?
A. Decreasing sodium intake
B. Weight loss program
C. Avoidance of excessive alcohol intake
D. Decreasing dietary fat intake
E. Regular exercise program
18. Human papilloma virus (HPV) infection has been
strongly associated with all the following cancers
EXCEPT:
A. Vulvar
B. Testicular
C. Anal
D. Cervical
E. Vaginal
19. The following drugs are effective treatments of
Salmonella typhi EXCEPT:
A. Chloramphenicol
B. Trimethoprim-sulfamethoxazole (Septrim)
C. Ampicillin
D. Ketoconazole
E. Ciprofloxacin
20. Which of the following is true regarding dengue fever?
A. The infectious agent involved in dengue fever
is a parasite similar to Plasmodium species
B. Fatalities are common with this disease even in
the absence of hemorrhagic fever
C. The mode of transmission is by the bite of
infected sand flies
D. Prevention depends on the use of vaccine and
post-exposure immune globulin
E. Onset is sudden and violent
MCQs 2008 Arab Board For Community Medicine Exam Paper 1 (70%)
Dr.AbdulQawi Almohamadi
Page 16 of 107
21. All of the following are associated with a higher
mortality risk from diseases EXCEPT:
A. Having a low socio-economic
B. Status being female
C. Having few friends
D. Moving from one place to the other frequently
E. Being divorced
22. Which agent is most likely responsible for a chronic
infection leading to cirrhosis or primary hepatocellular
carcinoma?
A. HAV
B. HBV
C. HCV
D. HDV
E. HIV
23. The administration of human immune globulin after
exposure to the Hepatitis B virus is an example of…
A. Secondary prevention
B. Health promotion
C. Passive immunity
D. Hypersensitivity
E. Cross reactivity
24. All the following have been identified as major risk
factors for Coronary Heart Disease, EXCEPT:
A. High serum cholesterol
B. Uncontrolled elevation in blood pressure
C. Cigarette smoking
D. Lack of daily vigorous exercise
E. Family history
25. Which of the following vaccines would most likely be
dangerous to a person with immunodeficiency?
A. Tetanus
B. Typhoid
C. Diphtheria
D. HBV
E. Measles
26. Routine antenatal care for normal pregnant women
should include all the following EXCEPT:
A. Weight checks
B. Blood pressure checks
C. Hemoglobin level tests
D. Urine examination
E. Electronic fetal monitoring
27. Of the following, the most effective method of family
planning is...
A. Contraceptive pills
B. Withdrawal
C. Safe period
D. Condom
E. Traditional methods
28. The value of alpha serves as protection against...
A. Type I error
B. Type II error
C. Type III error
D. False negative results
E. Inadequate sample size
Q 29 – 31 :
A count of cases found the following distribution of
observation: 1, 2, 2, 2, 3, 5, 5, 6,6,18.
29. The mean is :
A. 3
B. 4
C. 5
D. 6
E. 7
30. The median is :
A. 3
B. 4
C. 5
D. 6
E. 7
31. The mode is:
A. 6
B. 5
C. 4
D. 3
E. 2
32. A randomized clinical trial was designed to compare two
different treatment approaches for epileptic attacks. The
purpose of randomization in this study was to
A. Obtain treatment groups of similar size
B. Select a representative sample of patients for
study
C. Increase patient compliance with treatment
D. Decrease the likelihood that observed
differences in clinical outcome are due to
chance
E. Obtain treatment groups with comparable
baseline values
33. The purpose of the informed consent in a clinical trial is
to
A. Increase the patient's knowledge of possible
risks and benefits of treatment options
B. Increase the level of patient's participation
C. Minimize the effect of placebo
D. Decrease the likelihood of patient blinding to
treatment assignment
E. Prevent the occurrence of malpractice suits
MCQs 2008 Arab Board For Community Medicine Exam Paper 1 (70%)
Dr.AbdulQawi Almohamadi
Page 17 of 107
34. Breast feeding usually improves all the following
EXCEPT:
A. Uterine involution
B. Bonding
C. Infant nutrition
D. Infant immune system
E. Prevention of HIV infection
35. Of the following environmental hazards, which one is the
only exposure with a Dose-response effect?
A. Ionizing radiation
B. Noise
C. Heat
D. Lead
E. Carbon monoxide
36. The site of most frequent and serious contact with
environmental hazards is
A. Skin
B. Hands
C. Eyes
D. Gastrointestinal Tract
E. Lungs
Q37- 38:
Dr.Baha observed a rapid recovery in one of his Hepatitis
C patients who sat in a hospital solarium daily. A review
of the literature revealed no reported effects of the sun on
the convalescence of hepatitis patients, so Dr Baha
published a summary of this patients unusual recovery.
37. Which of the following study designs has Dr.Baha used?
A. Case report
B. Case-control
C. Environmental cohort
D. Clinical trial
E. Cross-sectional report
38. Dr Baha enlists the support of several gastroenterologists
and designs a study whereby Hepatitis C patients, upon
hospital admission are alternately assigned to receive
either 2 hours of sun exposure daily or no sun exposure.
Which of the following study designs has Dr Baha used?
A. Case report
B. Case-control
C. Environmental cohort
D. Clinical trial
E. Cross-sectional report
39. An indicator measuring short term outputs in Extended
Programs of Immunization (EPI) is
A. Number of refrigerators available in centers to
keep vaccines in the cold chain
B. Number of vaccination sessions
C. Coverage rates
D. Decreases incidence of measles
E. Number of trained health workers in EPI
40. The leading cause of fatal home injuries among children
under five is
A. Burning
B. Explosions
C. Poisoning
D. Falls
E. Drowning
41. The best method for treating water from a very turbid
source is ...
A. Alum coagulation
B. Rapid sand filtration
C. Activated charcoal
D. Ozonation.
E. Aeration
42. The pollutants globally known as Volatile Organic
Chemicals tend to concentrate in:
A. Stagnant water
B. Air
C. Fatty tissues
D. Edible fish tissue
E. Soil
43. All of the following cancers have been associated with
occupational exposure EXCEPT:
A. Bladder cancer
B. lung cancer
C. Hematopoietic cancer
D. Breast cancer
E. Liver cancer
44. Principals of Primary Health Care include all the
following items EXCEPT:
A. Multisectorial approach to health
B. Appropriate technology
C. Equity of resource
D. Distribution sanitation
E. Emphasis on prevention
45. Which one of the following is a true statement regarding
the "need" for medical care and the "demand" for
medical care?
A. They are different because patients need care
but physicians demand it
B. They are different because "need" is
professionally defined and "demand" is patient
generated
C. Need" usually surpasses "demand"
D. They are different because "need" takes cost
into consideration
E. They are the same when barriers to care are
minimal.
MCQs 2008 Arab Board For Community Medicine Exam Paper 1 (70%)
Dr.AbdulQawi Almohamadi
Page 18 of 107
46. Medical ethics are founded on all of the following basic
principles, EXCEPT:
A. Disclosure of information
B. Non-malfeasance
C. Beneficence.
D. Justice
E. Respect for patient's autonomy
47. Which statement characterizes families in which spouse
abuse occurs?
A. Husbands of battered women have higher
education than the ones who are not
B. Women who were abused as children are more
likely to be abused by their spouse
C. Women whose mothers suffered abuse are
more likely to be abused than those who did
not witness
D. In the Arab world, men who batter women
should expect to be prosecuted
E. Older women are more at risk of spouse abuse
than younger ones
48. Emotional disturbance of the post-partum
A. Are more common after the birth of a male
child
B. Are more common with increasing parity
C. Rarely last for more than 1 week
D. Occur in at least 20%of women
E. Can be predicted through psychometric tests in
pre-natal care
49. When debating whether providing or withholding
medical treatment is ethical, the LEAST important
consideration should be...
A. The indications for medical intervention
B. Expected quality of life after the treatment
C. Patient's preferences
D. Physician's preferences
E. Economic factors
50. The nutritional disease characterized by depression,
dermatitis and diarrhea is...
A. Ariboflavinosis
B. Beriberi
C. Scurvy
D. Pellagra
E. Kwashiorkor
51. Which of the following statements concerning iron
requirements is correct?
A. Requirements of women are not affected by
menopause
B. Requirements are the same in men and women
C. They increase during pregnancy
D. Can be satisfied by cow's milk
E. Iron requirements during pregnancy does not
vary by the pregnancy's trimester
52. In a double-blinded clinical trial concerning the treatment
of osteoarthritis, half of the patients received a new drug
and the other half received a pharmacologically inert
substance. Two thirds of the patients in the former group
and one third of the patients in the latter group reported
relief of symptoms. The patients' perceptions of
improvement on treatment with an inert substance is best
described as...
A. Intention to treat
B. Confounding effect
C. Placebo effect
D. Type II error
E. False positive results
53. No difference in the 5-year survival percentages is
observed in one small clinical trial, despite the fact that
the new treatment is very promising. Failure to detect a
benefit for the new treatment is best described as...
A. Observer bias
B. Placebo effect
C. Type I error
D. Type II error
E. Survivor bias
54. Routine chest X-rays will result in the early diagnosis of
a lung cancer case, with no impact on delaying the
eventual time of death. The longer apparent duration of
such a case can be attributed to the
A. High sensitivity of chest X-rays
B. High correlation
C. Lead-time bias
D. Length bias
E. Low prevalence of lung cancer
Q55-56:
During an 8-hour shift in a building, 30 employees (20
females and 10 males) visited the company physician
with complaints of nausea, vomiting, headache and
dizziness. All affected individuals responded to
supportive treatment and were sent home.
55. If 600 persons worked in the building, then the attack
rate was...
A. 3%
B. 5%
C. 10%
D. 20%
E. 30%
56. If 400 females and 200 males worked in the building, the
male to female risk ratio was...
A. 0.3
B. 0.5
C. 1
D. 2
E. 3
MCQs 2008 Arab Board For Community Medicine Exam Paper 1 (70%)
Dr.AbdulQawi Almohamadi
Page 19 of 107
Q57-59.:
A survey of all workers was conducted to determine
whether other persons were affected beyond those who
sought care at the physician's office. All 600 employees
were surveyed, and 400 questionnaires were completed
and returned. A total of 80 persons reported symptoms
consistent with the syndrome observed among workers
who sought medical attention.
57. The response rate to the questionnaire is nearest to...
A. 5.0%
B. 7.5%
C. 20.0%
D. 37.5%
E. 67.0%
58. Based on the survey data, the attack rate was...
A. 5.0%
B. 7.5%
C. 20.0%
D. 37.5%
E. 67.0%
59. In this case, the possibility of a secondary attack among
contacts would most likely indicate...
A. A point exposure
B. Vector transmission
C. Sexual transmission
D. Fecal-oral contamination
E. Person-to-person transmission
60. Which of the following is the best measure to estimate
the proportion of children newly entering school with
auditory defects?
A. Incidence rate
B. Prevalence
C. Risk ratio
D. Median survival
E. Case fatality
61. In a cohort study of hormone replacement therapy (HRT)
and risk of coronary artery disease (CAD), high-income
level is associated with both HRT and risk of CAD.
Which statement best describes the effect of "high
income" as a variable in this analysis?
A. Selection bias
B. Misclassification bias
C. Confounding
D. Ecological fallacy
E. Random error
62. In a cohort study of the relationship between dietary
supplementation and the occurrence of hip fractures, the
relative risk of hip fractures with supplementation was
0.5 with a 95% CI of 0.18 - 1.41. The correct
interpretation of this finding is that…
A. A significant association exists between
calcium supplementation and lower risk of hip
fracture.
B. The association between calcium
supplementation and risk of hip fracture should
not have been measured with the relative
risk.
C. The risk of hip fracture with calcium
supplementation is not statistically significant
from the risk.
D. One can conclude with 95% confidence that
calcium supplement protects against hip
fracture.
E. A Chi-square test should be performed to
assess the statistical significance of the
association.
Q63-66. :
A sea resort has a population of 99,000 which can be
divided into three age ranges: 25 - 44, 45-64 and 65 and
older. Each comprises one third of the population. In July
2003, 100 cases of hepatitis A occurred in the resort and
were traced to contaminated oysters. Of these 100 cases,
20 were between ages 25-44, 10 between ages 45-64
and five over the age of 64 ultimately proved fatal.
Before 2003 no cases had been reported among the
inhabitants of the resort.
63. What is the crude death rate (per 1000 residents) in the
resort?
A. 0.354
B. 1.01
C. 1.06
D. 35.4
E. Cannot be determined from the given data
64. What is the incidence of hepatitis A (per 1000) in July
2003?
A. 0.35
B. 1.01
C. 40
D. 350
E. Cannot be determined from the given data
65. What is the age specific mortality rate (per 10,000) for
residents over 64 years of age?
A. 1.52
B. 3.03
C. 3.54
D. 4.55
E. 6.06
66. What is the case fatality rate for hepatitis A?
A. 3.03 per 10,000
B. 3.54 per 10,000
C. 350 per 1000
D. per 1000
E. Cannot be determined from the given data
MCQs 2008 Arab Board For Community Medicine Exam Paper 1 (70%)
Dr.AbdulQawi Almohamadi
Page 20 of 107
67. Regarding statistical tests, which of the following is
NOT true?
A. Paired t-test is useful in the analysis of
qualitative data
B. The Student's t-test is used to compare two
independent sample means with an assumption
of equal
C. The z-score is the number of standard
deviations at which a value in a normally
distributed population lies away from the mean
D. Chi-square test is used in testing the
relationship between two proportions
E. The correlation coefficient is used to test the
relation between two continuous variables
68. Which of the following changes will decrease the width
of a confidence internal?
A. Increase in the level of confidence
B. Increase in subject-to-subject variation in
response variables
C. Increase in the sample size
D. Decrease in the precision of the estimate of the
population mean
E. Increase between the sample mean and the
population mean
Q69-72.:
Assume that you are evaluating a new screening test in a
population of 1000 people, of whom 75 are known cases
for the disorder under screening. Of the 75 afflicted
persons, 50 yield positive test results; 75 of the normal
people also have positive test results.
69. From this information, the sensitivity for the test is
A. 85%
B. 67%
C. 58%
D. 92%
E. Cannot be measured with the data provided
70. From this information, the specificity for the test is
A. 9%
B. 33%
C. 40%
D. 87%
E. 92%
71. From this information, the positive predictive value for
the test is
A. 60%
B. 40%
C. 20%
D. 10%
E. 97%
72. From this information, the negative predictive value for
the test is
A. 33%
B. 40%
C. 50%
D. 88%
E. 97%
73. Which one of the following diseases has not been linked
to cigarette smoking?
A. Carcinoma of the larynx
B. Hypertension
C. Abruption of placenta
D. Carcinoma of the colon
E. Alzheimer's disease
74. Of the following factors listed, which is the most
important factor in determining the success of a smoking
cessation program in an individual?
A. The desire of the patient to quit smoking
B. An effective pharmacologic agent
C. The inclusion of a behavior modification
component to the program
D. Physician's advice to quit smoking
E. Repeated office visits
75. Which of the following facts regarding colon cancer
screening is correct?
A. The US Task Force recommends only two
screening methods: fecal occult, blood testing
and colonoscopy.
B. Patients should be informed of the risk of
perforation or serious bleeding from
colonoscopy is about one/thousand procedures
in the community
C. Colonoscopy has been proved to reduce
mortality in large scale screening trials
D. Colon preparation for colonoscopy involves the
administration of two enemas before the
procedure
E. Fecal occult blood testing involves virtually no
preparation on the part of the patient
76. Which one of the following is NOT a risk factor for
breast carcinoma?
A. First degree relative with breast cancer
B. Nulliparity
C. Birth of first child after age 35
D. Early menarche
E. Breast feeding
Results
Positive
Negative
Total
Disease
50
25
75
Healthy
75
850
925
Total
125
875
1000
MCQs 2008 Arab Board For Community Medicine Exam Paper 1 (70%)
Dr.AbdulQawi Almohamadi
Page 21 of 107
77. Which of the following settings is NOT acceptable for
the delivery of bad news?
A. Physician's office
B. Quiet room in a hospital setting
C. Patient's home
D. Private room
E. Shared hospital room
78. An elevated triglyceride level is associated most closely
with which of the following problems?
A. Impaired glycemic control
B. Hyperthyroidism
C. Weight loss
D. Very Low Density Lipidemia (VLDL)
E. Elevated total serum cholesterol level
79. What is the mode of action of omeprazole?
A. H1 receptor angonist
B. H2 receptor antagonist
C. Proton pump inhibitor
D. Cytoprotection
E. Anticholinergic action
80. Which of the following statements regarding H. pylori is
false?
A. The acute infection is self limited and will
resolve without antibiotics
B. Organisms are found in human feces but not in
saliva
C. Income and socio-economic factors do not
influence prevalence
D. Approximately 25% of adults are colonized by
the age of 50 years
E. Aminoglycosides are indicated as first line of
treatment
81. Which of the following is the LEAST common
complaint in a victim of spousal abuse?
A. Back pain
B. Headache
C. Dyspareunia
D. Spousal abuse itself
E. Abdominal pain
82. Blood that is used for transfusion purposes is screened
routinely for serologic markers of all the following
diseases EXCEPT:
A. Hepatitis B virus
B. Herpes simplex virus
C. Syphilis
D. Cytomegalovirus
E. HIV
83. Which of the following is the most common site of
hospital-acquired infections?
A. Surgical wound
B. Respiratory tract
C. Skin
D. Urinary tract
E. Gastrointestinal tract
84. All the following are clearly associated with acute
episodes of asthma EXCEPT:
A. Emotional stress
B. Elevated ozone concentrations
C. Aspirin
D. Caffeinated beverages
E. Upper respiratory infection
85. The most important risk factor for developing
osteoporosis is
A. Smoking
B. Alcohol use
C. Lack of physical activity
D. Age
E. Obesity
86. Possible risk factors of the development of colon cancer
include all the following EXCEPT:
A. Irritable bowel syndrome
B. Diet rich in animal fat.
C. Inflammatory bowel disease.
D. Familial polyposis,
E. Uretrosigmoidostomy
87. A person working in hot environment and taking a lot of
water but no salt will develop a condition known as heat
A. Stroke
B. Syncope
C. Cramps
D. Exhaustion
E. Hyperpyrexia
88. Effects of greenhouse gases include all the following
EXCEPT:
A. Spread of insect born diseases to new areas
B. Natural weather disasters
C. Summers becoming hotter
D. Submersion of low-lying areas
E. Ozone depletion
89. Among the problems facing health manpower planning
in developing countries are the following EXCEPT:
A. Improper mix of different categories of staff
B. Low demand for nursing staff
C. Migration
D. Shortage in numbers
E. Faulty training curriculum
90. In postnatal care
A. One medical examination is enough
B. Breast-feeding promotion is crucial
C. It is advisable to continue health supervision
until the uterus returns to the pelvis
D. Exercise is contraindicated
E. Home visits by the health worker are advisable
for all mothers
MCQs 2008 Arab Board For Community Medicine Exam Paper 1 (70%)
Dr.AbdulQawi Almohamadi
Page 22 of 107
91. The marginal cost in the context of health care refers to
the cost of
A. Delivering one extra unit of services
B. Training and recruiting staff
C. Buildings and equipment
D. All daily activities
E. Care consumption
92. High parity may be associated with the following
EXCEPT:
A. Hypertensive disorders of pregnancy
B. Cancer of the cervix
C. Breast cancer
D. High risk of infant mortality
E. Gestational diabetes mellitus
93. The best indicator for the status of iron stores in the body
is...
A. hemoglobin concentration
B. Total iron binding capacity
C. Serum iron
D. Serum ferritin
E. Mean corpuscular hemoglobin concentration
94. Which is the process of day to day follow up of activities
to ensure that health programs are proceeding as
planned?
A. Evaluation
B. Surveillance
C. Monitoring
D. Implementation
E. Fixing priorities
95. In establishing communication with the patient, it is said
that the eyes are the most expressive part of the face.
They are the most useful organ for identifying which one
of the following
A. Anger
B. Fear
C. Sincerity
D. Depression
E. Disgust
96. The mental illness most likely to occur in young adults
(15-24) is…
A. Autism
B. Schizophrenia
C. Affective psychosis
D. Involutional melancholia
E. Agitated depression
97. Placing blame for one's own difficulties upon others is
considered as...
A. Displacement
B. Rationalization
C. Projection
D. Reaction formation
E. Obsession
98. Which of the following factors is a major factor in
increasing patient's compliance?
A. The absence of a psychiatric disorder
B. Having a chronic disease requiring long-term
treatment
C. The need for a significant behavioral change
D. Prescribing medicines with few side effects
E. Prescribing treatments with lower costs
99. The association between low birth weight and maternal
smoking during pregnancy can be studied by obtaining
smoking histories from women at the time of their
prenatal visit and correlating it with newborn's birth
weight. What type of study is this?
A. Clinical trial
B. Cross-sectional
C. Prospective cohort
D. Retrospective
E. Correlational
100. In order to determine the relationship between serum
levels of sodium and antidiuretic hormone (ADH) in
patients who have meningitis, which is the most
appropriate study design?
A. Repeated measurement of sodium and ADH in
a patient
B. Measurement of sodium and ADH in a set of
patients
C. Measurement of sodium in a set of patients and
of ADH in a different set of patients.
D. Measurement of ADH in a set of patients and a
set of controls.
E. All the above
101. All the following statements regarding the normal
(Gaussian) distribution are true EXCEPT:
A. The mean = median = mode .
B. Approximately 50 percent of observations are
greater than the mode.
C. Approximately 68 percent of observations falls
within1 standard deviation of the mean
D. The number of observations between O
and1standard deviation from the mean is the
same as the number between 1 and 2 standard
deviations from the mean.
E. The shape of the curve does not depend on the
value of the mean
102. Which of the following mechanism could discourage the
overutilization of health services?
A. Full health care coverage insurance plans
B. Deductibles and co-payment
C. Government supported health care delivery
system
D. Home health care plans
E. Hospice care
MCQs 2008 Arab Board For Community Medicine Exam Paper 1 (70%)
Dr.AbdulQawi Almohamadi
Page 23 of 107
103. Which one of the following pesticides is capable of
causing pulmonary fibrosis?
A. Organophosphates
B. Parathion
C. Halogenated hydrocarbons
D. Paraquat
E. Organochlorines
104. The object of disinfection of water is to …
A. Reduce the bacterial population
B. Destroy fecal material
C. Prevent bacterial toxin formation
D. Limit growth of pathogenic
E. Bacteria remove turbidity
105. Which of the following agents are most resistant to
disinfection with chlorinated water?
A. Hepatitis virus species
B. Schistosomes
C. Coliform organisms
D. Cysts of E. histolytica
E. Enteric bacteria
106. Chemical food poisoning associated with preparations
added to raw meat to preserve the red color is most
commonly due to...
A. Antimony
B. Arsenic
C. Cadmium
D. Nicotinic acid
E. Nitrates
107. Which of the following general effects would be most
characteristic of an ascent to an altitude of 4,000 meters?
A. Deterioration of voluntary muscle control
B. Impairment of ocular balance
C. Impairment of hearing
D. Increased night oxygen requirements
E. Visual acuity less than 50%of normal
108. Caisson disease is the result of
A. Swelling of synovial tissues
B. Carbon dioxide saturation in the blood
C. Air in the intestinal tract
D. Nitrogen bubbles in the blood stream
E. Oxygen bubbles in soft tissues.
109. In DOTS, all the following are true EXCEPT:
A. This method can solve the problem of defaults
in treatment.
B. It is a program with a high-cost effectiveness.
C. It is designed to be used by trained doctors.
D. Can prevent the development of drug
resistance.
E. Has up to 96% cure rate.
110. The longitudinal research approach is characterized by
A. The same group being studied over extended
periods of time.
B. Different groups being studied over extended
periods of time.
C. Individuals being studied at different intervals
throughout their life cycle.
D. Groups of individuals of overlapping ages
being tested periodically in similar dimensions.
E. Different dimensions of behavior being
examined at different stages of development to
determine short-range effects of longitudinal
analysis.
111. The bacteria that are involved in nosocomial infections
are transmitted most often by:
A. Airborne matter.
B. Fomites.
C. Exposure to a common source.
D. Indwelling catheters.
E. Direct contact via hands.
112. Which one of the following problems is a documented
hazard related to the use of oral contraceptives?
A. Breast cancer
B. Vaginal cancer
C. Osteoporosis
D. Thromboembolism
E. Brain tumor
113. Patients indicate their satisfaction with treatment by
responding to a question with four options: very
dissatisfied, dissatisfied, satisfied, and very satisfied
which type of scale is this?
A. Nominal scale
B. Ordinal scale
C. Internal scale
D. Ratio scale
E. Categorical scale
114. State the most appropriate statistical test to analyze data
comparing the serum cholesterol before and after
ingestion of a hamburgers in a sample of persons.
A. Student's t-test .
B. Multiple regression.
C. Paired t-test .
D. Correlation study.
E. Chi-square
MCQs 2008 Arab Board For Community Medicine Exam Paper 1 (70%)
Dr.AbdulQawi Almohamadi
Page 24 of 107
115. The difference between impairment and disability is that
A. Impairment is socially defined.
B. Disability is socially defined.
C. Disability is medically defined.
D. Impairment is not permanent.
E. Disability is permanent.
116. The ability to produce a live-born baby is defined as
A. Fertility
B. Fecundity
C. Fertility rate
D. Crude birth rate
E. Total fertility rate
117. "Perceived susceptibility, severity, obstacles and
benefits" are determinants of behavior modification in
which theoretical model?
A. Health belief model
B. Social learning model
C. Theory of planned behavior
D. Stages of change theory
E. Triad model
118. The most consistent parental behavior found in cases of
child abuse is...
A. Inconsistency in discipline
B. Employment of harsh mental punishment
C. Rejection
D. Parents treating the child as they were treated
by their parents
E. Correction
119. A worker diagnosed with silicosis is at highest risk of
developing:
A. Lung cancer
B. Tuberculosis
C. Emphysema
D. Bronchiectasis
E. Arteriosclerosis
120. The most frequent cancer site in Middle-East women is
A. Lung
B. Breast
C. Colorectal
D. Uterus
E. Ovary
************************
GOOD
2014 Exam Paper (70%)- MCQs (Duration 150 minutes)
Dr.AbdulQawi Almohamadi
Page 25 of 107
MCQs 2014
1. Epidemiology can be defined as the study of
A. The etiology of disease in humans
B. The determinants of disease frequency in
humans
C. The distribution and determinants of disease
frequency in humans.
D. The patterns of health care organization and
financing.
E. The-distribution of disease in case of an
epidemic
2. The association between low birth weight and maternal
smoking during pregnancy can be studied by obtaining
smoking histories from women at the time of their
prenatal visit and then subsequently correlating birth
weight with smoking histories. What type of study is
this?
A. Clinical trial
B. Cross-sectional
C. Prospective (cohort)
D. Retrospective case-control
E. None of the above.
3. Which of the following measures is used frequently as a
denominator to calculate the rate of a disease?
A. Number of cases observed
B. Number of new cases observed.
C. Number of asymptomatic cases.
D. Person-years of observation.
E. Persons lost to follow-up.
4. In the study of the cause of a disease, the essential
difference between an experimental study and an
observational study is that in the experimental
investigation
A. The study is prospective.
B. The study is retrospective.
C. The study and control groups are of equal size
D. The study and control groups are selected on
the basis of history of exposure to the
suspected causal factor
E. The investigators determine who is and who is
not exposed to the suspected causal factors
Questions (5 – 6):
To determine whether prenatal exposure to tobacco
smoke is a cause of undescended testes in newborns, the
mothers of 100 newborns with undescended testes and
100 newborns whose TESTES bad descended, were
questioned about smoking habits during pregnancy. The
study revealed an odds ratio of 2.6 associated with
exposure to smoke, with 95% confidence intervals from
(1.1 to 5.3) .
5. Which of the following statement is true?
A. The odds ratio could be falsely elevated by the
inclusion of infants whose testes were
descended (but retractile) in the case group
(misclassification bias).
B. The odds ratio could be falsely elevated by
recall bias if parents of affected infants were
more likely to remember or report their
exposures.
C. Because the cases are newborns, but the
exposure data came from their mothers, this is
not a true case-control study.
D. Since the study was not blind, it is impossible
to rule out a placebo effect
E. None of the above
6. Which of the following statements is true?
A. The results provide no evidence that maternal
cigarette smoking is associated with
undescended testes in the offspring
B. If the study results are accurate, they suggest
that a baby boy whose mother smoked is about
2.6 times as likely to be born with testes
undescended as a baby boy whose mother did
not smoke
C. The fact that the confidence interval excludes 1
indicates that P>.05
D. The 90 percent confidence interval for these
results would probably include 1.0
E. None of the above
7. In order to determine the relationship between serum
levels of sodium and antidiuretic hormone (ADH) in
patients, who have meningitis, the most appropriate
study design would be :
A. Repeated measurement of sodium and ADH in
a patient
B. Measurement of sodium and ADH in a set of
patients
C. Measurement of sodium in a set of patients and
of ADH in a different set of patients.
D. Measurement of ADH in a set of patients and a
set of controls.
E. None of the above.
8. Assume that the annual death rate from lung cancer for
British doctors is 160 per 100,000 among heavy smokers
compared with 8 per 100,000 among non smokers. What
is the relative risk of dying of lung cancer for smokers
compared with nonsmokers?
A. 152
B. 20
C. 19
D. 8
E. None of the above
2014 Exam Paper (70%)- MCQs (Duration 150 minutes)
Dr.AbdulQawi Almohamadi
Page 26 of 107
P= c (1 + r)ⁿ
Chance =1 - (1- 0.1)³⁰
9. Randomization is a procedure used for assignment or
allocation of subjects to treatment and control groups in
experimental studies. Randomization ensures:
A. That assignment occurs by chance
B. That treatment and control groups are alike in
all respects EXCEPT treatment
C. That bias in observations is eliminated
D. That placebo effects are eliminated
E. None of the above
10. In study of the cause of lung cancer, patients who disease
were matched with controls by age, sex, place of
residence and social class. The frequency of cigarette
smoking was then compared in the two groups. What
type of study was this?
A. Prospective (cohort)
B. Retrospective ( case- control)
C. Clinical trial
D. Historical prospective
E. None of the above
Questions 11-14;
During 1999, 200 newly diagnosed cases of diabetes
occurred in city X, which has a population of 10,000
(estimates on July 1, 1999). At the beginning of the year,
there were a total of 800 patients with diabetes in the
city. During the year, 40 patients died of the
complications of diabetes.
11. What was the average annual incidence of diabetes
during 1999?
A. 4 per 1000
B. 80 per 1000
C. 100 per 1000
D. 200 per 1000
E. 20 per 1000
12. What was the approximate point prevalence of diabetes
on January 1, 1999?
A. 20 per 1,000
B. 80 per 1,000
C. 200 per 1,000
D. 800 per 1.000
E. 1000 per 1,000
13. What was the prevalence of diabetes on December
31,1999:
A. 16 per 1,000
B. 20 per 1,000
C. 80 per 1,000
D. 96 per 1,000
E. 1000 per 1,000
14. What was the mortality due diabetes during 1999?
A. 0.4 per 1,000
B. 1.6 per 1,000
C. 2.0 per 1,000
D. 4.0 per 1,000
E. 9.6 per 1,000
15. The sampling method in which each individual of the
total group has an equal chance of being selected in the
sample is :
A. Systematic sampling.
B. Paired sampling.
C. Simple random sampling.
D. Stratified sampling.
E. Cluster sampling.
16. A sample of 1,000 people includes 120 who are hearing
impaired and 50 who are diabetic. If the number who are
both diabetic and hearing impaired is 6, then.
A. Diabetes and hearing impairment appear to be
independent.
B. Diabetics appear to be protected from hearing
impairment.
C. Diabetics appear to be at greater risk of hearing
impairment.
D. There is an interaction between diabetes and
hearing impairment
E. There is not sufficient information to state any
of the above.
17. A measures of the amount of variation of a set of values
about the mean of the set is the:
A. Regression coefficient
B. Standard error of the mean
C. Standard deviation
D. Range
E. Correlation coefficient
18. Suppose that each time an individual receives pooled
blood products, there is a 10 percent chance of that
person developing a cytomegalovirus (CMV) infection.
If an individual receives pooled blood products on 30
occasions, what is his or her chance of developing
cytomegalovirus.
A. 300 percent
B. 3 percent
C. 30 percent
D. 95.8 percent
E. 10 percent
19. All the following statements regarding the normal
(Gaussian) distribution are true EXCEPT:
A. The mean = median =mode
B. Approximately 50 percent of observations are
greater than mode.
C. Approximately 68 percent of observations falls
within 1 standard deviation of the mean
D. The number of observations between 0 and 1
standard deviation from the mean is the same
as the number between 1 and 2 standard
deviations from the mean.
E. The shape of the curve does not depend on the
value of the mean.
2014 Exam Paper (70%)- MCQs (Duration 150 minutes)
Dr.AbdulQawi Almohamadi
Page 27 of 107
20. Which of the following statements concerning statistical
inference is correct?
A. If the p value = 0.05 then there is a 95 percent
probability that the results didn't occur by
chance
B. The null Hypothesis generally states that there
is a difference between the groups
C. If the p value is sufficiently high, the null
hypothesis is not rejected
D. Knowledge of the sampling method is not
important in determining Statistical
significance.
E. None of the above.
21. The probability of being born with condition A is 0.10
and the probability of being born with condition B is
0.50 If conditions A and B are independent, what is the
probability of being born with either condition A or
condition B (or both)?
A. 0.05
B. 0.40
C. 0.50
D. 0.55
E. 0.60
22. In nine families surveyed, the numbers of children per
family were 4, 5, 2, 2, 4, 3, 2, 1, 7, The mean, median,
and mode numbers of children per family respectively
are:
A. 3,4.2,3
B. 3,3.4,2
C. 3,3,2
D. 2,3.5,3
E. None of the above
23. -------------- not available--------------
24. -------------- not available--------------
25. -------------- not available--------------
26. The type of statistical test that can best be employed in
comparison of systolic blood pressure in independent
sampling of pregnant and non pregnant women..
A. Chi-square analysis
B. Students t-test
C. Analysis variance
D. Paired t-test
E. Linear regression:
Q. 27-29.:
Patients who have had proven myocardial infarctions MI are
compared with matched controls without a history of infarction as
the basis of electrocardiograms. The table below shows the
27. The sensitivity is :
A. (92)
B. (84)
C. (16)
D. (73)
E. (95)
28. The false positives proportion is :
A. (92)
B. (84)
C. (16)
D. (73)
E. (95)
29. The positive predictive value is
A. (92)
B. (84)
C. (16)
D. (73)
E. (95)
30. The best available source of information to estimate the
incidence of the meningococcal meningitis is :
A. Death certificate.
B. Household surveys.
C. Cancer registry.
D. Directorate of public health department
records.
E. Life insurance companies.
31. Health information system uses information related to:
A. Efficiency of health service
B. Effectiveness of preventive programs
C. Extent of coverage of health programs
D. Health personnel shortages
E. All of the above
32. The most important statement about health information
is:
A. It must affect decision making
B. It must be precise
C. It must be computerized
D. It must be comprehensive
E. It must be easily retrieved
Results
Abnormal
ECG
Normal
ECG
Total
MI
1100
100
1200
Healthy
400
2100
2500
Total
1500
2200
3700
A + B A
B
2014 Exam Paper (70%)- MCQs (Duration 150 minutes)
Dr.AbdulQawi Almohamadi
Page 28 of 107
33. Technology has increased the cost of health care
because:
A. It has decreased the number of health workers
B. It has provided better diagnosis
C. It has improved information cost
D. It often provides care but not total cure
E. It often leads to better paid administrators
34. Which of the following mechanism could discourage
over utilization of health services
A. Full health care coverage insurance plans
B. Deductibles and coinsurance
C. Government supported health care delivery
system
D. Home health care plans
E. Hospice care
35. If the prevalence of protein Calorie ( PC) under nutrition
with overt signs and symptoms in a country is 2% , then:
A. (PC) under nutrition is not public health
problem in that country
B. Subclinical (PC) under nutrition is low.
C. The incidence of (PC) under nutrition is high
D. The case fatality is high
E. The prevalence of (PC) under nutrition is low
compared with other countries
36. To conduct a comprehensive nutritional assessment, the
epidemiologist usually start with: (the starting data
source):
A. Community cross-sectional survey
B. Routine health data
C. Nutritional surveillance
D. Case-control study
E. Ecological study
37. Air pollution:
A. Has an effect only in the immediate
neighborhood of the source.
B. Is rapidly cleared by temperature inversions.
C. Is associated with increased mortality from all
causes and not just respiratory diseases.
D. Humidity and mist has no synergistic effect on
air pollution
E. Is more important in the causation of lung
cancer than tobacco smoking.
38. An important indicator of air pollution is :
A. Chlorine
B. Fluorine
C. Sulphur dioxide
D. Carbon dioxide
E. Hydrogen
39. Which one of the following pesticides is capable of
causing pulmonary fibrosis?
A. Organophosphates
B. Parathion
C. Halogenated hydrocarbons
D. Paraquat
E. Organochlorines
40. The Dose of Gama-Ray Radiation received from a fixed
source is:
A. Inversely proportional to the square of distance
B. Directly proportional to the distance
C. Directly proportional to the square of the
distance
D. Expressed as 3 times the product of the
distance and time.
E. Expressed as the sum of the squares of the
distance and rate of exposure.
41. The best method for treating water from a very turbid
source is:
A. Aluminum coagulation
B. Rapid sand filtration
C. Activated charcoal
D. Ozonation
E. Aeration
42. Testing For Albuminoidal Nitrogen And Nitrites Would
Best Be Utilized In:
A. Air pollution control
B. Sewage processing
C. Pasteurization of milk
D. Water purification
E. Food processing
43. Sulfate of Aluminum is commonly utilized in
A. Milk purification
B. Decreasing air pollution
C. Prevention of radiation hazard
D. Sewage disposal
E. Water purification
44. The object of disinfection of water is to:
A. Reduce the bacterial population
B. Destroy bacterial population
C. Prevent bacterial toxin formation
D. Limit growth of pathogenic bacteria
E. None of the above
45. Which of following has been LEAST implicated in
diseases spread by milk and milk products?
A. Cream
B. Certified milk
C. Butter
D. Butter milk
E. Ice cream
46. DDT is an effective insecticide that operates as a:
A. Repellant
B. CNS poison
C. Bactericide
D. Stomach poison
E. Fertility inhibitor
2014 Exam Paper (70%)- MCQs (Duration 150 minutes)
Dr.AbdulQawi Almohamadi
Page 29 of 107
47. Which of the following is the most resistant to water
disinfection.
A. Virus of infections hepatitis
B. Schistosomiasis
C. Coliform organisms
D. Cysts of E. histolytica
E. Enteric bacteria
48. An electrostatic precipitator is best used in:
A. Air pollution control
B. Sewage disposal
C. Pasteurization of milk
D. Water purification
E. Arthropod-vector control
49. The cheapest method of refuse disposal is:
A. Incineration
B. Dumping at sea
C. Reduction Sanitary
D. Sanitary fill
E. Feeding to animals
50. The Phosphatase test is associated with:
A. Water purification
B. Pasteurization of milk
C. Sewage disposal
D. Air pollution control
E. None of the above
51. Chemical food poisoning associated with preparations
added to raw meat to preserve the red color, is most
commonly due to :
A. Antimony
B. Arsenic
C. Cadmium
D. Nicotinic acid
E. Nitrates
52. Which of the following general effects would be most
character of an ascent to an altitude of 12.000 feet?
A. Deterioration of voluntary muscle control
B. Impairment of ocular balance
C. Impairment of hearing
D. Oxygen would be required at night
E. Visual acuity less than 50% of normal
53. Which of the following does not apply to carbon
monoxide?
A. Always present in the exhaust from internal
combustion engines
B. Odorless
C. Tasteless
D. Has a lesser affinity for hemoglobin than
oxygen
E. Non-irritating
54. Which of the following dusts predispose to tuberculosis?
A. Asbestos
B. Zinc
C. Silica
D. Graphite
E. None of the above
55. Major purpose of medical service in industry:
A. To prevent job related injury or illness
B. To provide emergency medical aid
C. Effective employee placement
D. To maintain good health for the total personnel
by health promotion and health education.
E. All of the above
56. A patient with headache, nausea, vomiting, fatigue,
jaundice, hepatomegaly and oliguria, may have an
occupational disease caused by:
A. Lead
B. Mercury
C. Benzol
D. Carbon tetrachloride
E. Hydrogen sulfide
57. Damage to reproductive cells may occur as a result of
occupational exposure to
A. Chromium.
B. Lead
C. Mercury
D. Zinc
E. Carbon monoxide
58. Caisson disease is the result of:
A. Swelling of synovial tissues
B. Carbon dioxide saturation in the blood
C. Air in the abdomen
D. Nitrogen bubbles in the blood stream
E. Oxygen bubbles in soft tissues.
59. Which of the following effects has not been
demonstrated to be the result of industrial noise?
A. Increased peristalsis
B. Increased blood pressure
C. Decreased gastric secretion
D. Irregular heart action
E. Fatigue
60. Which of the following causes the highest loss of work
days:
A. Circulatory diseases
B. Bone and joint diseases
C. Digestive disorders
D. Allergic disorders
E. Nutritional diseases
2014 Exam Paper (70%)- MCQs (Duration 150 minutes)
Dr.AbdulQawi Almohamadi
Page 30 of 107
61. Persons at an increased risk of bladder cancer include
workers in all the following industries EXCEPT:
A. Manufacture of rubber
B. Dyeing of textiles
C. Manufacture of paints
D. Manufacture of steel
E. Printing
62. Which of the following substances is causally associated
with pneumoconiosis?
A. Sulfur oxides
B. Nitrogen oxides
C. Oil fumes
D. Dust particles
E. Cigarette smoke
63. The major environmental source of lead absorbed in the
human blood stream is:
A. Air
B. Water
C. Lead-based paint
D. Food
E. None of the above
64. Prolonged exposure to polyvinyl chlorides in production
is association with each of the following EXCEPT:
A. Acroosteolysis
B. Raynaud disease
C. Lung disease
D. Angiosarcoma of the liver
E. Scleroderma
65. The most common asbestos-related tumor in human is
A. Bronchogenic carcinoma
B. Carcinoma of the colon
C. Pleural mesothelioma
D. Peritoneal mesothelioma
E. Pericardial mesothelioma
66. Ozone depletion due to air pollution is controlled by:
A. Using lead free Fuel!
B. Reduction of world production of Freon
C. Filtration of exhaust
D. Wide use of electric energy instead of Fuels
E. Banning the use of refrigerators.
67. The development of neuropathic symptoms is associated
with chronic exposure to all the following substance
EXCEPT:
A. Mercury
B. Lead
C. Arsenic
D. Sulfur dioxide
E. Nitrous oxide
68. The most prevalent mental disorder in young children is
A. Autism
B. Mental retardation
C. Behavioral problem
69. ---------- Unavailable …………….
70. ---------- Unavailable …………….
71. ---------- Unavailable …………….
72. The most prevalent psychiatric disorder among opiate
addicts undergoing treatment is
A. Schizophrenic
B. Depression
C. Alcoholism
D. Antisocial
E. Mania
73. In DOTS all the following are true EXCEPT:
A. Can solve the problem of defaults
B. Is a program with a high cost effectiveness
C. Is designed to be used by trained doctors
D. Can prevent the development of drug resistance
E. Has a up to 96% cure rate
74. B.C.G. vaccination:
A. Is given to all tuberculin positive cases in
endemic areas
B. Is given by intramuscular injection at deltoid
region
C. Is not recommended to be given to newly born
babies
D. Can give a partial or complete immunity for up
to 12 years
E. Can cause superlative lymphadenitis in 10% of
the cases
75. Brucella species can:
A. Survive direct sunlight for four days
B. Stand boiling for few hours
C. Stand freezing for several weeks
D. Be transmitted by yogurt made from sheep
milk
E. Easily transmitted from a patient to his direct
contacts
76. In the transmission of cholera
A. Direct spread among contacts is very important
during epidemics
B. Is rare in Europe because of cold climate
C. The highest rate is usually among young adults
during epidemics
D. Asymptomatic cases are commoner among
classical biotype
E. Long term carries are common
2014 Exam Paper (70%)- MCQs (Duration 150 minutes)
Dr.AbdulQawi Almohamadi
Page 31 of 107
77. A 11 years people was diagnosis to have Meningococcal
meningitis, the most important control measure in the
school is
A. Mass vaccination
B. Chemoprophylaxis by rifampicin
C. Close surveillance of contacts.
D. Good ventilation of classes
E. Improve the sanitation of the school.
78. All the following are true in typhoid fever EXCEPT:
A. In every outbreak there should be a human
carrier.
B. Fecal carries are usually associated with gall
bladder disease.
C. Vaccination by TAB (typhoid and paratyphoid
A&B) is superior to that by Ty21a.
D. A negative Widal test might be more useful in
exclusion the disease.
E. The usual incubation period is 10-14 days.
79. In Crimean- cango hemorrhagic fever?
A. The disease is limited to the Africa and
Crimean.
B. The mortality rate is 2-5%.
C. The reservoir is among ticks.
D. Transmission by mosquitoes
E. There are no subclinical cases.
80. In Rift Valley Fever all the following are true EXCEPT:
A. Mammals are the reservoir.
B. Has a low mortality rate among the human.
C. The vector is mites.
D. Has recently cross the red sea to the Yemen
and Saudi Arabia.
E. Can cause hemorrhagic manifestation.
81. The aims for Control Diarrhea Disease (CDD) program
are the following EXCEPT:
A. Epidemiologic surveillance for epidemics of
the disease
B. Initiating an ORT room is all PHC centers
C. Training mothers for proper and scientific use
of antibiotics
D. Promotion of breast feeding
E. Maintaining acute statistics
82. The following can increase the possibility of sexual
transmission of HIV infection EXCEPT:
A. Accompanied ulcerative STD
B. Very early infection
C. Late presentation of AIDS
D. Circumcision
E. Rectal sex
83. Chlamydia species can cause the following diseases in
man EXCEPT :
A. Non-specific cervicitis
B. Trachoma
C. Lymph granuloma veneered
D. Chancroid
E. Psittacosis
84. An influenza pandemic is usually expected when:
A. There is an antigenic shift in type B
B. There is an antigenic drift in type B
C. The isolated strain is Hong Kong type C
D. There is an antigenic shift in type A
E. There is a direct transmission from swine to
man
85. Amoebic dysentery is differentiated from bacillary
dysentery by having more tendency to be:
A. Transmitted directly from man to man
B. Associated with prostration
C. Associated with tenesmus
D. Presented as sporadic cases
E. Easily transmitted by water
86. The following are true association in cancer EXCEPT:
A. Cigarette smoking and bladder cancer
B. Sexual promiscuity and cervical cancer
C. Vinyl chloride and liver angiosarcoma
D. Early menopause and breast cancer
E. Nullipara and ovarian cancer
87. In viral hepatitis type C:
A. Screening test is not yet available
B. Vertical transmission is common
C. Immunization is readily available
D. The incubation period is 5-7 weeks
E. Animal reservoir is important
88. Transplacental transmission can occur in all the
following disease EXCEPT:
A. Rubella
B. Cytomegalovirus infection
C. Toxoplasmosis
D. Hepatitis A infection
E. Syphilis
89. The first step in conducting an epidemic investigation is
to:
A. Determine the number of cases
B. Calculate the incubation period
C. Determine the population at risk
D. Verify the diagnosis
E. Collect appropriate samples
2014 Exam Paper (70%)- MCQs (Duration 150 minutes)
Dr.AbdulQawi Almohamadi
Page 32 of 107
90. The best measure of obstetric care quality in a
community is:
A. Crude death rate
B. Infant mortality rate
C. Abortion rate
D. Perinatal mortality rate
E. None of the above
91. Which of the following is most characteristic of the
abuser of the elderly?
A. Encourages of the elderly person to speak his
or her mind
B. Has cared for the elderly person for some time
C. Lives separately from the abused person
D. Takes immaculate care of the house and the
abused person
E. Volunteered to take care of the abused person
92. The longitudinal research approach is characterized by:
A. The same group being studied over extended
periods of time
B. Different groups being studied over extended
periods of time
C. Individual being studied at different intervals
throughout their life cycle
D. Groups of individual of overlapping ages being
tested periodically in similar dimensions
E. Different dimensions of behavior being
examined at different stages of development to
determine short-range effects of longitudinal
analysis
93. Death from overdose is a strong possibly with all of the
following drugs EXCEPT:
A. Alcohol
B. Narcotics
C. Sedative-hypnotics
D. Psychedelics (including marijuana)
E. Phencyclidine
94. A confounding variable may relate closely in time in the
of disease but:
A. Is not necessary for its occurrence
B. Is one of several causal agents
C. Is more difficult to measure than other
variables
D. Exacerbates rather than causes the disease
E. Can't be alleviated through disease prevention
programs.
95. Which of the following is characteristic of a single
exposure, common vehicle outbreak?
A. Frequent secondary cases
B. Increasing severity with increasing age
C. Explosive
D. Cases include in both people who have been
exposed and those didn't exposed.
E. All of the above
96. In the study of an outbreak of an infectious disease
plotting an epidemic curve is useful because
A. It helps determine what type of outbreak (e.g.
single-space, person-to-person) has occurred
B. It shows whether herd Immunity has occurred
C. It helps determine the median incubation
period
D. a and c
E. a,b and c
97. Which of the following is/are a good index of the
severity of short-term acute disease:
A. Cause-specific death rate
B. 5-year survival
C. Case-fatality rate
D. Standardized mortality ratio
E. None of the above
98. Ecologic fallacy refers to:
A. Assessing exposure in large groups rather than
in many small groups
B. Assessing outcome in large groups rather than
in many small groups
C. Ascribing the characteristics of a group to
every individual in that group
D. Examining correlations of exposure and
outcomes rather than time trends
E. Failure to examine temporal relations between
exposures and outcomes
99. Diabetes mellitus
A. Is easily defined for clinical and
epidemiological purposes
B. Prevalence increases with age
C. Dose not affect life expectancy
D. Usually causes death through its specific
complications
E. Occurs only in industrialized, sedentary,
Western populations
100. The risk of coronary heart disease from smoking
cigarettes :
A. Shows the greatest relative risk in the younger
age groups
B. Appears to be irreversible in those who stop
smoking
C. Accounts for less smoking related deaths than
does lung cancer
D. Has a greater relative risk than does smoking
for lung cancer
E. Operates in populations with very low serum
cholesterol levels.
101. The following are Modifiable Risk Factors of
Hypertension EXCEPT:
A. Obesity
B. Genetic Factors
C. Stress
D. Excess salt intake
E. Oral contraceptive use
Arab Board examination for Community Medicine.pdf
Arab Board examination for Community Medicine.pdf
Arab Board examination for Community Medicine.pdf
Arab Board examination for Community Medicine.pdf
Arab Board examination for Community Medicine.pdf
Arab Board examination for Community Medicine.pdf
Arab Board examination for Community Medicine.pdf
Arab Board examination for Community Medicine.pdf
Arab Board examination for Community Medicine.pdf
Arab Board examination for Community Medicine.pdf
Arab Board examination for Community Medicine.pdf
Arab Board examination for Community Medicine.pdf
Arab Board examination for Community Medicine.pdf
Arab Board examination for Community Medicine.pdf
Arab Board examination for Community Medicine.pdf
Arab Board examination for Community Medicine.pdf
Arab Board examination for Community Medicine.pdf
Arab Board examination for Community Medicine.pdf
Arab Board examination for Community Medicine.pdf
Arab Board examination for Community Medicine.pdf
Arab Board examination for Community Medicine.pdf
Arab Board examination for Community Medicine.pdf
Arab Board examination for Community Medicine.pdf
Arab Board examination for Community Medicine.pdf
Arab Board examination for Community Medicine.pdf
Arab Board examination for Community Medicine.pdf
Arab Board examination for Community Medicine.pdf
Arab Board examination for Community Medicine.pdf
Arab Board examination for Community Medicine.pdf
Arab Board examination for Community Medicine.pdf
Arab Board examination for Community Medicine.pdf
Arab Board examination for Community Medicine.pdf
Arab Board examination for Community Medicine.pdf
Arab Board examination for Community Medicine.pdf
Arab Board examination for Community Medicine.pdf
Arab Board examination for Community Medicine.pdf
Arab Board examination for Community Medicine.pdf
Arab Board examination for Community Medicine.pdf
Arab Board examination for Community Medicine.pdf
Arab Board examination for Community Medicine.pdf
Arab Board examination for Community Medicine.pdf
Arab Board examination for Community Medicine.pdf
Arab Board examination for Community Medicine.pdf
Arab Board examination for Community Medicine.pdf
Arab Board examination for Community Medicine.pdf
Arab Board examination for Community Medicine.pdf
Arab Board examination for Community Medicine.pdf
Arab Board examination for Community Medicine.pdf
Arab Board examination for Community Medicine.pdf
Arab Board examination for Community Medicine.pdf
Arab Board examination for Community Medicine.pdf
Arab Board examination for Community Medicine.pdf
Arab Board examination for Community Medicine.pdf
Arab Board examination for Community Medicine.pdf
Arab Board examination for Community Medicine.pdf
Arab Board examination for Community Medicine.pdf
Arab Board examination for Community Medicine.pdf
Arab Board examination for Community Medicine.pdf
Arab Board examination for Community Medicine.pdf
Arab Board examination for Community Medicine.pdf
Arab Board examination for Community Medicine.pdf
Arab Board examination for Community Medicine.pdf
Arab Board examination for Community Medicine.pdf
Arab Board examination for Community Medicine.pdf
Arab Board examination for Community Medicine.pdf
Arab Board examination for Community Medicine.pdf
Arab Board examination for Community Medicine.pdf
Arab Board examination for Community Medicine.pdf
Arab Board examination for Community Medicine.pdf
Arab Board examination for Community Medicine.pdf
Arab Board examination for Community Medicine.pdf
Arab Board examination for Community Medicine.pdf
Arab Board examination for Community Medicine.pdf
Arab Board examination for Community Medicine.pdf
Arab Board examination for Community Medicine.pdf

Weitere ähnliche Inhalte

Was ist angesagt? (20)

Association causation
Association causationAssociation causation
Association causation
 
DESCRIPTIVE EPIDEMIOLOGY
DESCRIPTIVE EPIDEMIOLOGYDESCRIPTIVE EPIDEMIOLOGY
DESCRIPTIVE EPIDEMIOLOGY
 
Descriptive epidemiology
Descriptive epidemiologyDescriptive epidemiology
Descriptive epidemiology
 
Multiple Choice Questions - Biostatistics
Multiple Choice Questions - BiostatisticsMultiple Choice Questions - Biostatistics
Multiple Choice Questions - Biostatistics
 
Secondary attack rates
Secondary attack ratesSecondary attack rates
Secondary attack rates
 
Odds ratio
Odds ratioOdds ratio
Odds ratio
 
Mcq in diaster Magement
Mcq in diaster Magement Mcq in diaster Magement
Mcq in diaster Magement
 
Lecture of epidemiology
Lecture of epidemiologyLecture of epidemiology
Lecture of epidemiology
 
Cohort study - basics
Cohort study - basicsCohort study - basics
Cohort study - basics
 
6. Calculate samplesize for cohort studies
6. Calculate samplesize for cohort studies6. Calculate samplesize for cohort studies
6. Calculate samplesize for cohort studies
 
5. Case control
5. Case control5. Case control
5. Case control
 
Community Medicine BCQs (Primary health care)
Community Medicine BCQs (Primary health care)Community Medicine BCQs (Primary health care)
Community Medicine BCQs (Primary health care)
 
What is a health system?
What is a health system?What is a health system?
What is a health system?
 
Cross sectional study by Dr Abhishek Kumar
Cross sectional study by Dr Abhishek KumarCross sectional study by Dr Abhishek Kumar
Cross sectional study by Dr Abhishek Kumar
 
Global disease burden
Global disease burdenGlobal disease burden
Global disease burden
 
Health Promotion- Models
Health Promotion- ModelsHealth Promotion- Models
Health Promotion- Models
 
Epidemeology
EpidemeologyEpidemeology
Epidemeology
 
Sample size calculation for cohort studies
Sample size calculation for cohort studies Sample size calculation for cohort studies
Sample size calculation for cohort studies
 
Surveilance
SurveilanceSurveilance
Surveilance
 
Public health surveillance
Public health surveillancePublic health surveillance
Public health surveillance
 

Ähnlich wie Arab Board examination for Community Medicine.pdf

1. Toxic substances can be described by their ability to cause un
1.  Toxic substances can be described by their ability to cause un1.  Toxic substances can be described by their ability to cause un
1. Toxic substances can be described by their ability to cause unsandibabcock
 
1.Questions 1 and 2 are based on the following informationYou.docx
1.Questions 1 and 2 are based on the following informationYou.docx1.Questions 1 and 2 are based on the following informationYou.docx
1.Questions 1 and 2 are based on the following informationYou.docxchristiandean12115
 
NUTRITIONAL EPIDEMIOLOGY END OF SEMESTER EXAM.docx
NUTRITIONAL EPIDEMIOLOGY END OF SEMESTER EXAM.docxNUTRITIONAL EPIDEMIOLOGY END OF SEMESTER EXAM.docx
NUTRITIONAL EPIDEMIOLOGY END OF SEMESTER EXAM.docxkihembopamelah
 
(Practical Applications of Epidemiology)Please choose the most
(Practical Applications of Epidemiology)Please choose the most(Practical Applications of Epidemiology)Please choose the most
(Practical Applications of Epidemiology)Please choose the mosthirstcruz
 
Multiple health problems in elderly peoplepage 950Ex.docx
Multiple health problems in elderly peoplepage 950Ex.docxMultiple health problems in elderly peoplepage 950Ex.docx
Multiple health problems in elderly peoplepage 950Ex.docxgilpinleeanna
 
Evidence based population health screening
Evidence based population health screeningEvidence based population health screening
Evidence based population health screeningmeducationdotnet
 
Community health nursing examination part i answer key
Community health nursing examination part i answer keyCommunity health nursing examination part i answer key
Community health nursing examination part i answer keyryanmejia
 
Analytical epidemiology
Analytical  epidemiologyAnalytical  epidemiology
Analytical epidemiologyb_bhushan
 
Central Line-associated Bloodstream Infections.Walden Universi
Central Line-associated Bloodstream Infections.Walden UniversiCentral Line-associated Bloodstream Infections.Walden Universi
Central Line-associated Bloodstream Infections.Walden UniversiMaximaSheffield592
 
Introduction to Clinical Epidemiology (401173) FINAL ASSIGNMENT
Introduction to Clinical Epidemiology (401173) FINAL ASSIGNMENTIntroduction to Clinical Epidemiology (401173) FINAL ASSIGNMENT
Introduction to Clinical Epidemiology (401173) FINAL ASSIGNMENThildredzr1di
 
American Guideline Pneumonia.pdf
American Guideline Pneumonia.pdfAmerican Guideline Pneumonia.pdf
American Guideline Pneumonia.pdfNataliaSaezDuarte
 
01 guia carat uso_racional_de_atb
01 guia carat uso_racional_de_atb01 guia carat uso_racional_de_atb
01 guia carat uso_racional_de_atbjano231054
 
-APA-825words-No plagiarism, will check with turnitin
-APA-825words-No plagiarism, will check with turnitin-APA-825words-No plagiarism, will check with turnitin
-APA-825words-No plagiarism, will check with turnitinjolleybendicty
 
Journal club - Disease progression in hemodynamically stable patients present...
Journal club - Disease progression in hemodynamically stable patients present...Journal club - Disease progression in hemodynamically stable patients present...
Journal club - Disease progression in hemodynamically stable patients present...Farooq Khan
 
Test bank pathophysiology, concepts of human disease by matthew sorenson
Test bank pathophysiology, concepts of human disease by matthew sorensonTest bank pathophysiology, concepts of human disease by matthew sorenson
Test bank pathophysiology, concepts of human disease by matthew sorensonsolahar
 

Ähnlich wie Arab Board examination for Community Medicine.pdf (20)

1. Toxic substances can be described by their ability to cause un
1.  Toxic substances can be described by their ability to cause un1.  Toxic substances can be described by their ability to cause un
1. Toxic substances can be described by their ability to cause un
 
1.Questions 1 and 2 are based on the following informationYou.docx
1.Questions 1 and 2 are based on the following informationYou.docx1.Questions 1 and 2 are based on the following informationYou.docx
1.Questions 1 and 2 are based on the following informationYou.docx
 
NUTRITIONAL EPIDEMIOLOGY END OF SEMESTER EXAM.docx
NUTRITIONAL EPIDEMIOLOGY END OF SEMESTER EXAM.docxNUTRITIONAL EPIDEMIOLOGY END OF SEMESTER EXAM.docx
NUTRITIONAL EPIDEMIOLOGY END OF SEMESTER EXAM.docx
 
EPIDEMIOLOGY .pptx
EPIDEMIOLOGY  .pptxEPIDEMIOLOGY  .pptx
EPIDEMIOLOGY .pptx
 
Fundamental of nursing review questions
Fundamental of nursing review questionsFundamental of nursing review questions
Fundamental of nursing review questions
 
(Practical Applications of Epidemiology)Please choose the most
(Practical Applications of Epidemiology)Please choose the most(Practical Applications of Epidemiology)Please choose the most
(Practical Applications of Epidemiology)Please choose the most
 
Multiple health problems in elderly peoplepage 950Ex.docx
Multiple health problems in elderly peoplepage 950Ex.docxMultiple health problems in elderly peoplepage 950Ex.docx
Multiple health problems in elderly peoplepage 950Ex.docx
 
Evidence based population health screening
Evidence based population health screeningEvidence based population health screening
Evidence based population health screening
 
Community health nursing examination part i answer key
Community health nursing examination part i answer keyCommunity health nursing examination part i answer key
Community health nursing examination part i answer key
 
Analytical epidemiology
Analytical  epidemiologyAnalytical  epidemiology
Analytical epidemiology
 
Central Line-associated Bloodstream Infections.Walden Universi
Central Line-associated Bloodstream Infections.Walden UniversiCentral Line-associated Bloodstream Infections.Walden Universi
Central Line-associated Bloodstream Infections.Walden Universi
 
Introduction to Clinical Epidemiology (401173) FINAL ASSIGNMENT
Introduction to Clinical Epidemiology (401173) FINAL ASSIGNMENTIntroduction to Clinical Epidemiology (401173) FINAL ASSIGNMENT
Introduction to Clinical Epidemiology (401173) FINAL ASSIGNMENT
 
Ats neumonia
Ats neumoniaAts neumonia
Ats neumonia
 
Epidemiology
EpidemiologyEpidemiology
Epidemiology
 
American Guideline Pneumonia.pdf
American Guideline Pneumonia.pdfAmerican Guideline Pneumonia.pdf
American Guideline Pneumonia.pdf
 
Cohort study
Cohort studyCohort study
Cohort study
 
01 guia carat uso_racional_de_atb
01 guia carat uso_racional_de_atb01 guia carat uso_racional_de_atb
01 guia carat uso_racional_de_atb
 
-APA-825words-No plagiarism, will check with turnitin
-APA-825words-No plagiarism, will check with turnitin-APA-825words-No plagiarism, will check with turnitin
-APA-825words-No plagiarism, will check with turnitin
 
Journal club - Disease progression in hemodynamically stable patients present...
Journal club - Disease progression in hemodynamically stable patients present...Journal club - Disease progression in hemodynamically stable patients present...
Journal club - Disease progression in hemodynamically stable patients present...
 
Test bank pathophysiology, concepts of human disease by matthew sorenson
Test bank pathophysiology, concepts of human disease by matthew sorensonTest bank pathophysiology, concepts of human disease by matthew sorenson
Test bank pathophysiology, concepts of human disease by matthew sorenson
 

Mehr von Dr. AbdulQawi Almohamadi

Mehr von Dr. AbdulQawi Almohamadi (9)

Dengue fever guidelines therapy
Dengue fever guidelines therapy Dengue fever guidelines therapy
Dengue fever guidelines therapy
 
How to select a statistical test?
How to select a statistical test?How to select a statistical test?
How to select a statistical test?
 
Golden rules of correlation & regression
Golden rules of correlation & regression Golden rules of correlation & regression
Golden rules of correlation & regression
 
الاحصاء و تحليل البيانات
الاحصاء و تحليل البياناتالاحصاء و تحليل البيانات
الاحصاء و تحليل البيانات
 
Principles of Scientific Research
Principles of Scientific Research Principles of Scientific Research
Principles of Scientific Research
 
Effective risk management in healthcare practice-
Effective risk management in healthcare practice-Effective risk management in healthcare practice-
Effective risk management in healthcare practice-
 
FOCUS PDCA
FOCUS  PDCA FOCUS  PDCA
FOCUS PDCA
 
قياس الجودة بواسطة وضع المؤشرات
قياس الجودة  بواسطة وضع المؤشراتقياس الجودة  بواسطة وضع المؤشرات
قياس الجودة بواسطة وضع المؤشرات
 
تصميم بحوث النظم الصحية
تصميم بحوث النظم الصحيةتصميم بحوث النظم الصحية
تصميم بحوث النظم الصحية
 

Kürzlich hochgeladen

Jogeshwari ! Call Girls Service Mumbai - 450+ Call Girl Cash Payment 90042684...
Jogeshwari ! Call Girls Service Mumbai - 450+ Call Girl Cash Payment 90042684...Jogeshwari ! Call Girls Service Mumbai - 450+ Call Girl Cash Payment 90042684...
Jogeshwari ! Call Girls Service Mumbai - 450+ Call Girl Cash Payment 90042684...Anamika Rawat
 
Andheri East ^ (Genuine) Escort Service Mumbai ₹7.5k Pick Up & Drop With Cash...
Andheri East ^ (Genuine) Escort Service Mumbai ₹7.5k Pick Up & Drop With Cash...Andheri East ^ (Genuine) Escort Service Mumbai ₹7.5k Pick Up & Drop With Cash...
Andheri East ^ (Genuine) Escort Service Mumbai ₹7.5k Pick Up & Drop With Cash...Anamika Rawat
 
Call Girls in Lucknow Just Call 👉👉7877925207 Top Class Call Girl Service Avai...
Call Girls in Lucknow Just Call 👉👉7877925207 Top Class Call Girl Service Avai...Call Girls in Lucknow Just Call 👉👉7877925207 Top Class Call Girl Service Avai...
Call Girls in Lucknow Just Call 👉👉7877925207 Top Class Call Girl Service Avai...adilkhan87451
 
Kollam call girls Mallu aunty service 7877702510
Kollam call girls Mallu aunty service 7877702510Kollam call girls Mallu aunty service 7877702510
Kollam call girls Mallu aunty service 7877702510Vipesco
 
Coimbatore Call Girls in Thudiyalur : 7427069034 High Profile Model Escorts |...
Coimbatore Call Girls in Thudiyalur : 7427069034 High Profile Model Escorts |...Coimbatore Call Girls in Thudiyalur : 7427069034 High Profile Model Escorts |...
Coimbatore Call Girls in Thudiyalur : 7427069034 High Profile Model Escorts |...chennailover
 
Call Girls Madurai Just Call 9630942363 Top Class Call Girl Service Available
Call Girls Madurai Just Call 9630942363 Top Class Call Girl Service AvailableCall Girls Madurai Just Call 9630942363 Top Class Call Girl Service Available
Call Girls Madurai Just Call 9630942363 Top Class Call Girl Service AvailableGENUINE ESCORT AGENCY
 
Call Girl In Pune 👉 Just CALL ME: 9352988975 💋 Call Out Call Both With High p...
Call Girl In Pune 👉 Just CALL ME: 9352988975 💋 Call Out Call Both With High p...Call Girl In Pune 👉 Just CALL ME: 9352988975 💋 Call Out Call Both With High p...
Call Girl In Pune 👉 Just CALL ME: 9352988975 💋 Call Out Call Both With High p...chetankumar9855
 
Independent Call Girls Service Mohali Sector 116 | 6367187148 | Call Girl Ser...
Independent Call Girls Service Mohali Sector 116 | 6367187148 | Call Girl Ser...Independent Call Girls Service Mohali Sector 116 | 6367187148 | Call Girl Ser...
Independent Call Girls Service Mohali Sector 116 | 6367187148 | Call Girl Ser...karishmasinghjnh
 
Russian Call Girls Lucknow Just Call 👉👉7877925207 Top Class Call Girl Service...
Russian Call Girls Lucknow Just Call 👉👉7877925207 Top Class Call Girl Service...Russian Call Girls Lucknow Just Call 👉👉7877925207 Top Class Call Girl Service...
Russian Call Girls Lucknow Just Call 👉👉7877925207 Top Class Call Girl Service...adilkhan87451
 
Call Girls Jaipur Just Call 9521753030 Top Class Call Girl Service Available
Call Girls Jaipur Just Call 9521753030 Top Class Call Girl Service AvailableCall Girls Jaipur Just Call 9521753030 Top Class Call Girl Service Available
Call Girls Jaipur Just Call 9521753030 Top Class Call Girl Service AvailableJanvi Singh
 
Call Girls Service Jaipur {9521753030 } ❤️VVIP BHAWNA Call Girl in Jaipur Raj...
Call Girls Service Jaipur {9521753030 } ❤️VVIP BHAWNA Call Girl in Jaipur Raj...Call Girls Service Jaipur {9521753030 } ❤️VVIP BHAWNA Call Girl in Jaipur Raj...
Call Girls Service Jaipur {9521753030 } ❤️VVIP BHAWNA Call Girl in Jaipur Raj...khalifaescort01
 
Call Girls Kolkata Kalikapur 💯Call Us 🔝 8005736733 🔝 💃 Top Class Call Girl Se...
Call Girls Kolkata Kalikapur 💯Call Us 🔝 8005736733 🔝 💃 Top Class Call Girl Se...Call Girls Kolkata Kalikapur 💯Call Us 🔝 8005736733 🔝 💃 Top Class Call Girl Se...
Call Girls Kolkata Kalikapur 💯Call Us 🔝 8005736733 🔝 💃 Top Class Call Girl Se...Namrata Singh
 
Low Rate Call Girls Bangalore {7304373326} ❤️VVIP NISHA Call Girls in Bangalo...
Low Rate Call Girls Bangalore {7304373326} ❤️VVIP NISHA Call Girls in Bangalo...Low Rate Call Girls Bangalore {7304373326} ❤️VVIP NISHA Call Girls in Bangalo...
Low Rate Call Girls Bangalore {7304373326} ❤️VVIP NISHA Call Girls in Bangalo...Sheetaleventcompany
 
Top Quality Call Girl Service Kalyanpur 6378878445 Available Call Girls Any Time
Top Quality Call Girl Service Kalyanpur 6378878445 Available Call Girls Any TimeTop Quality Call Girl Service Kalyanpur 6378878445 Available Call Girls Any Time
Top Quality Call Girl Service Kalyanpur 6378878445 Available Call Girls Any TimeCall Girls Delhi
 
Call Girl in Indore 8827247818 {LowPrice} ❤️ (ahana) Indore Call Girls * UPA...
Call Girl in Indore 8827247818 {LowPrice} ❤️ (ahana) Indore Call Girls  * UPA...Call Girl in Indore 8827247818 {LowPrice} ❤️ (ahana) Indore Call Girls  * UPA...
Call Girl in Indore 8827247818 {LowPrice} ❤️ (ahana) Indore Call Girls * UPA...mahaiklolahd
 
VIP Hyderabad Call Girls Bahadurpally 7877925207 ₹5000 To 25K With AC Room 💚😋
VIP Hyderabad Call Girls Bahadurpally 7877925207 ₹5000 To 25K With AC Room 💚😋VIP Hyderabad Call Girls Bahadurpally 7877925207 ₹5000 To 25K With AC Room 💚😋
VIP Hyderabad Call Girls Bahadurpally 7877925207 ₹5000 To 25K With AC Room 💚😋TANUJA PANDEY
 
Top Rated Hyderabad Call Girls Erragadda ⟟ 9332606886 ⟟ Call Me For Genuine ...
Top Rated  Hyderabad Call Girls Erragadda ⟟ 9332606886 ⟟ Call Me For Genuine ...Top Rated  Hyderabad Call Girls Erragadda ⟟ 9332606886 ⟟ Call Me For Genuine ...
Top Rated Hyderabad Call Girls Erragadda ⟟ 9332606886 ⟟ Call Me For Genuine ...chandars293
 
Models Call Girls In Hyderabad 9630942363 Hyderabad Call Girl & Hyderabad Esc...
Models Call Girls In Hyderabad 9630942363 Hyderabad Call Girl & Hyderabad Esc...Models Call Girls In Hyderabad 9630942363 Hyderabad Call Girl & Hyderabad Esc...
Models Call Girls In Hyderabad 9630942363 Hyderabad Call Girl & Hyderabad Esc...GENUINE ESCORT AGENCY
 
Call Girls Mysore Just Call 8250077686 Top Class Call Girl Service Available
Call Girls Mysore Just Call 8250077686 Top Class Call Girl Service AvailableCall Girls Mysore Just Call 8250077686 Top Class Call Girl Service Available
Call Girls Mysore Just Call 8250077686 Top Class Call Girl Service AvailableDipal Arora
 

Kürzlich hochgeladen (20)

Jogeshwari ! Call Girls Service Mumbai - 450+ Call Girl Cash Payment 90042684...
Jogeshwari ! Call Girls Service Mumbai - 450+ Call Girl Cash Payment 90042684...Jogeshwari ! Call Girls Service Mumbai - 450+ Call Girl Cash Payment 90042684...
Jogeshwari ! Call Girls Service Mumbai - 450+ Call Girl Cash Payment 90042684...
 
Andheri East ^ (Genuine) Escort Service Mumbai ₹7.5k Pick Up & Drop With Cash...
Andheri East ^ (Genuine) Escort Service Mumbai ₹7.5k Pick Up & Drop With Cash...Andheri East ^ (Genuine) Escort Service Mumbai ₹7.5k Pick Up & Drop With Cash...
Andheri East ^ (Genuine) Escort Service Mumbai ₹7.5k Pick Up & Drop With Cash...
 
Call Girls in Lucknow Just Call 👉👉7877925207 Top Class Call Girl Service Avai...
Call Girls in Lucknow Just Call 👉👉7877925207 Top Class Call Girl Service Avai...Call Girls in Lucknow Just Call 👉👉7877925207 Top Class Call Girl Service Avai...
Call Girls in Lucknow Just Call 👉👉7877925207 Top Class Call Girl Service Avai...
 
Kollam call girls Mallu aunty service 7877702510
Kollam call girls Mallu aunty service 7877702510Kollam call girls Mallu aunty service 7877702510
Kollam call girls Mallu aunty service 7877702510
 
Call Girls in Gagan Vihar (delhi) call me [🔝 9953056974 🔝] escort service 24X7
Call Girls in Gagan Vihar (delhi) call me [🔝  9953056974 🔝] escort service 24X7Call Girls in Gagan Vihar (delhi) call me [🔝  9953056974 🔝] escort service 24X7
Call Girls in Gagan Vihar (delhi) call me [🔝 9953056974 🔝] escort service 24X7
 
Coimbatore Call Girls in Thudiyalur : 7427069034 High Profile Model Escorts |...
Coimbatore Call Girls in Thudiyalur : 7427069034 High Profile Model Escorts |...Coimbatore Call Girls in Thudiyalur : 7427069034 High Profile Model Escorts |...
Coimbatore Call Girls in Thudiyalur : 7427069034 High Profile Model Escorts |...
 
Call Girls Madurai Just Call 9630942363 Top Class Call Girl Service Available
Call Girls Madurai Just Call 9630942363 Top Class Call Girl Service AvailableCall Girls Madurai Just Call 9630942363 Top Class Call Girl Service Available
Call Girls Madurai Just Call 9630942363 Top Class Call Girl Service Available
 
Call Girl In Pune 👉 Just CALL ME: 9352988975 💋 Call Out Call Both With High p...
Call Girl In Pune 👉 Just CALL ME: 9352988975 💋 Call Out Call Both With High p...Call Girl In Pune 👉 Just CALL ME: 9352988975 💋 Call Out Call Both With High p...
Call Girl In Pune 👉 Just CALL ME: 9352988975 💋 Call Out Call Both With High p...
 
Independent Call Girls Service Mohali Sector 116 | 6367187148 | Call Girl Ser...
Independent Call Girls Service Mohali Sector 116 | 6367187148 | Call Girl Ser...Independent Call Girls Service Mohali Sector 116 | 6367187148 | Call Girl Ser...
Independent Call Girls Service Mohali Sector 116 | 6367187148 | Call Girl Ser...
 
Russian Call Girls Lucknow Just Call 👉👉7877925207 Top Class Call Girl Service...
Russian Call Girls Lucknow Just Call 👉👉7877925207 Top Class Call Girl Service...Russian Call Girls Lucknow Just Call 👉👉7877925207 Top Class Call Girl Service...
Russian Call Girls Lucknow Just Call 👉👉7877925207 Top Class Call Girl Service...
 
Call Girls Jaipur Just Call 9521753030 Top Class Call Girl Service Available
Call Girls Jaipur Just Call 9521753030 Top Class Call Girl Service AvailableCall Girls Jaipur Just Call 9521753030 Top Class Call Girl Service Available
Call Girls Jaipur Just Call 9521753030 Top Class Call Girl Service Available
 
Call Girls Service Jaipur {9521753030 } ❤️VVIP BHAWNA Call Girl in Jaipur Raj...
Call Girls Service Jaipur {9521753030 } ❤️VVIP BHAWNA Call Girl in Jaipur Raj...Call Girls Service Jaipur {9521753030 } ❤️VVIP BHAWNA Call Girl in Jaipur Raj...
Call Girls Service Jaipur {9521753030 } ❤️VVIP BHAWNA Call Girl in Jaipur Raj...
 
Call Girls Kolkata Kalikapur 💯Call Us 🔝 8005736733 🔝 💃 Top Class Call Girl Se...
Call Girls Kolkata Kalikapur 💯Call Us 🔝 8005736733 🔝 💃 Top Class Call Girl Se...Call Girls Kolkata Kalikapur 💯Call Us 🔝 8005736733 🔝 💃 Top Class Call Girl Se...
Call Girls Kolkata Kalikapur 💯Call Us 🔝 8005736733 🔝 💃 Top Class Call Girl Se...
 
Low Rate Call Girls Bangalore {7304373326} ❤️VVIP NISHA Call Girls in Bangalo...
Low Rate Call Girls Bangalore {7304373326} ❤️VVIP NISHA Call Girls in Bangalo...Low Rate Call Girls Bangalore {7304373326} ❤️VVIP NISHA Call Girls in Bangalo...
Low Rate Call Girls Bangalore {7304373326} ❤️VVIP NISHA Call Girls in Bangalo...
 
Top Quality Call Girl Service Kalyanpur 6378878445 Available Call Girls Any Time
Top Quality Call Girl Service Kalyanpur 6378878445 Available Call Girls Any TimeTop Quality Call Girl Service Kalyanpur 6378878445 Available Call Girls Any Time
Top Quality Call Girl Service Kalyanpur 6378878445 Available Call Girls Any Time
 
Call Girl in Indore 8827247818 {LowPrice} ❤️ (ahana) Indore Call Girls * UPA...
Call Girl in Indore 8827247818 {LowPrice} ❤️ (ahana) Indore Call Girls  * UPA...Call Girl in Indore 8827247818 {LowPrice} ❤️ (ahana) Indore Call Girls  * UPA...
Call Girl in Indore 8827247818 {LowPrice} ❤️ (ahana) Indore Call Girls * UPA...
 
VIP Hyderabad Call Girls Bahadurpally 7877925207 ₹5000 To 25K With AC Room 💚😋
VIP Hyderabad Call Girls Bahadurpally 7877925207 ₹5000 To 25K With AC Room 💚😋VIP Hyderabad Call Girls Bahadurpally 7877925207 ₹5000 To 25K With AC Room 💚😋
VIP Hyderabad Call Girls Bahadurpally 7877925207 ₹5000 To 25K With AC Room 💚😋
 
Top Rated Hyderabad Call Girls Erragadda ⟟ 9332606886 ⟟ Call Me For Genuine ...
Top Rated  Hyderabad Call Girls Erragadda ⟟ 9332606886 ⟟ Call Me For Genuine ...Top Rated  Hyderabad Call Girls Erragadda ⟟ 9332606886 ⟟ Call Me For Genuine ...
Top Rated Hyderabad Call Girls Erragadda ⟟ 9332606886 ⟟ Call Me For Genuine ...
 
Models Call Girls In Hyderabad 9630942363 Hyderabad Call Girl & Hyderabad Esc...
Models Call Girls In Hyderabad 9630942363 Hyderabad Call Girl & Hyderabad Esc...Models Call Girls In Hyderabad 9630942363 Hyderabad Call Girl & Hyderabad Esc...
Models Call Girls In Hyderabad 9630942363 Hyderabad Call Girl & Hyderabad Esc...
 
Call Girls Mysore Just Call 8250077686 Top Class Call Girl Service Available
Call Girls Mysore Just Call 8250077686 Top Class Call Girl Service AvailableCall Girls Mysore Just Call 8250077686 Top Class Call Girl Service Available
Call Girls Mysore Just Call 8250077686 Top Class Call Girl Service Available
 

Arab Board examination for Community Medicine.pdf

  • 1. MCQS AND ASSAY COVER The Arab Board for Medical Specializations Scientific Council for Family And community Medicine Board Certification In community Medicine (PART I) 2007, 2008, (FROM 2014, TO 2021). Dr.AbdulQawi Almohamadi abdulqawiqaid.wordpress.com
  • 2. Contents COVER__________________________________________________________________ 1 Contents ________________________________________________________________ 2 MCQs 2007______________________________________________________________ 3 MCQs 2008_____________________________________________________________ 14 MCQs 2014_____________________________________________________________ 25 MCQs 2015_____________________________________________________________ 35 Long Questions 2015 _____________________________________________________ 44 MCQs 2016_____________________________________________________________ 45 Long Questions 2016 _____________________________________________________ 53 MCQs 2017_____________________________________________________________ 54 Long Questions 2017 _____________________________________________________ 65 MCQS 2018 ____________________________________________________________ 66 Long Questions 2018 _____________________________________________________ 77 MCQs April 2019 ________________________________________________________ 78 Long Questions April 2019_________________________________________________ 87 MCQs October 2019 ______________________________________________________ 88 Long Questions October 2019 ______________________________________________ 97 MCQs January 2021______________________________________________________ 98 Long Questions January 2021______________________________________________107 Answers Exam January 2021 ______________________________________________107
  • 3. [2007 arab board for community medicine exam paper one (70%)- mcqs.] Dr.AbdulQawi Almohamadi Page 3 of 107 MCQs 2007 1. Epidemiology can be best defined as the study of A. The etiology of disease in humans. B. The distribution and determinants of health-related states or events. C. The patterns of organization and financing of health care systems. D. The frequency of causes of death. E. The prevention and control of infectious diseases. 2. A slender 50-year-old with fair skin is being seen for a routine pap smear. She is postmenopausal and in good health with no known medical problems. She leads a sedentary life mostly indoors. Which of the following preventive measures is most appropriate for her? A. Electrocardiogram. B. Discussion of aspirin therapy. C. Discussion of estrogen replacement therapy. D. Instructions to use skin protection from UV light. E. None of the above. 3. For a man aged 40 to 50 years, with no known medical problems, which of the following screening tests should routinely be performed? A. Urinalysis. B. Sigmoidoscopy. C. Cholesterol. D. Prostate specific antigen. E. None of the above. 4. Which of the following is a leading potential cause of death for a healthy child aged 2 years, for which a preventive measure is available and which should be addressed with the parents? A. Lead toxicity B. Motor vehicle accident C. Tuberculosis D. Pneumonia E. Influenza Q5-6: A pharmaceutical company has developed a new treatment A for a rare disorder that is associated with a high risk of death. It conducted a randomized trial which included 200 subjects; half were assigned the active treatment, and the other half placebo. Thirty of the subjects receiving treatment died during the course of follow-up; forty of those receiving placebo did. 5. The relative risk of mortality for drug A compared to placebo is: A. 30/100 B. 40/100 C. 30/40 D. 30/200 E. 40-30/ 200 6. The statistical test for the effect of drug A has a P-value (2- Sided) = 0.027. Which of the following statements are correct interpretations about this statistical result? A. The probability that the null hypothesis is true is 2.7%. B. If in fact drug A does not affect mortality, the probability of observing a positive result is 2.7%. C. If in fact drug A does affect mortality, the probability of observing a positive result is 2.7%. D. If in fact drug A does affect mortality, the probability of observing a negative result is 2.7%. E. None of the above Q7-8: A Case-control study was performed to determine the association between cigarette smoking and acute myocardial infarction (MI). Cases were patients with MI admitted to a single hospital, and controls were patients hospitalized at the same hospital without a diagnosis of MI. Trained interviewers then asked all subjects about their prior smoking history. Assume that smoking does increase the risk of MI. 7. What characteristic is NOT appropriate for the "control" group? A. Controls should be disease-free. B. Controls should come from the same population from which cases were selected. C. Controls should have the same exposure as cases. D. Restrictions applied to cases should also apply to controls E. Controls should receive full information about the aims of the study 8. The most specific type of bias potentially found in Case- Control studies is the... A. Measurement bias B. Random bias C. Recall bias D. Study cost E. Loss to follow-up Results Drugs Placebo Total Died 30 40 70 Living 70 60 130 Total 100 100 200
  • 4. [2007 arab board for community medicine exam paper one (70%)- mcqs.] Dr.AbdulQawi Almohamadi Page 4 of 107 9. The preferred method for handling dropouts or non- compliance with treatment in a randomized trial is to... A. Analyze data according to the treatment actually received by the patients, not to how they were randomized. B. Analyze data as in answer (a), but control for baseline differences between groups in prognostic factors. C. Analyze data as though the patients randomized to a given treatment actually received that treatment, even though they may not have received that treatment. D. Analyze outcomes of patients who actually complete the course as intended. E. Analyze outcomes following the treatment received or those based on treatments to which patients were randomized, both are good approaches as long as blinding technique were correctly used. 10. A flight carried 98 passengers, 47 of whom reported suffering from gastrointestinal illness within four days of the flight. All the following statements regarding the attack rate are true, EXCEPT: A. The attack rate is the incidence rate of gastrointestinal illness among the flight passengers. B. The attack rate is an appropriate incidence rate for a specific group of individuals, under limited circumstances. C. 0.48 D. 0.12 per day. E. The attack rate may underestimate the real incidence of disease in this group if some mild cases are missed. 11. The functions of Public Health surveillance include all the following EXCEPT: A. Data collection. B. Data analysis. C. Interpretation of reported data. D. Dissemination of results. E. Disease control action in response to outbreak notification 12. The safest method of malaria prevention is: A. Chemoprophylaxis. B. Environmental control. C. Combination of drugs and insecticides. D. Biological control. E. Genetic methods. 13. Diseases that may affect the fetus include all, EXCEPT: A. Toxoplasmosis B. Rubella C. Tuberculosis D. Cytomegalovirus E. HSV 14. Which of the following is the most effective method for family planning? A. Oral contraceptive pills. B. Withdrawal. C. Safe period counts. D. Condom use. E. Traditional methods. 15. Breast feeding can improve all the following, EXCEPT: A. Uterine involution. B. Bonding. C. Infant nutrition. D. Infant immune system. E. Prevention of HIV infection. 16. Of the following environmental hazards, the only one which causes a non-threshold exposure is ... A. Ionizing radiation. B. Noise. C. Heat. D. Lead contamination. E. Carbon monoxide poisoning. 17. The diagnostic power of a test is reflected by its A. Sensitivity. B. Specificity. C. Predictive value. D. Attributable risk. E. Validity. 18. Which one is the most Cost-Beneficial technology in improving public health in developing nations? A. Safe water supply. B. Oral rehydration solutions. C. Hospitals in all communities with at least 1,000 population. D. One doctor for 500 population. E. Control of industrial pollution. 19. Folic acid supplementation is important in the prevention of A. Byssinosis. B. Intrauterine growth retardation. C. Pellagra. D. Neural tube defects. E. Preeclampsia. 20. All of the following cancers have been associated with occupational exposure, EXCEPT: A. Bladder cancer. B. Lung cancer. C. Hematopoietic cancers. D. Breast cancer. E. Liver cancer.
  • 5. [2007 arab board for community medicine exam paper one (70%)- mcqs.] Dr.AbdulQawi Almohamadi Page 5 of 107 21. How are numerical summaries of clinical measurements affected by a frequency distribution that is skewed to the right? A. The mean and the median are equal. B. The mean is larger than the median. C. The median is larger than the mean. D. The mode is larger than the median. E. The mean and the median are larger than the mode. 22. The vector of Leishmaniasis is... A. Culex fatigans. B. Anopheles mosquito. C. Phlebotomus sand-fly. D. Aedes Egypti. E. Culex mosquito. 23. Risk factors of gout include all the following, EXCEPT: A. Hyperuricemia. B. Alcohol consumption. C. Lead exposure. D. Female gender. E. Use of diuretics. 24. A person working in hot environment and taking a lot of water but no salt will develop a condition known as: A. Heat stroke. B. Heat syncope. C. Heat cramps. D. Heat exhaustion. E. Heat hyperpyrexia. 25. Farmers are at risk of all the following, EXCEPT: A. Malignant pustule. B. Hydaditosis. C. Toxic effects of pesticides. D. Pulmonary anthrax. E. Chronic skin changes. 26. Health care workers are at risk for developing which of the following occupational diseases? A. Silicosis. B. Bilharziasis. C. Byssinosis. D. Bagatosis. E. Hepatitis B infection. 27. The marginal cost in the context of health care refers to: A. The cost of delivering one extra unit of services. B. The cost of training and recruiting staff. C. The cost of buildings and equipment. D. The cost of all daily activities. E. The cost on the consumers of care. 28. Basic resources for providing optimal health care include all the following EXCEPT: A. Doctors, nurses and other health personnel. B. Health centers, hospitals and clinics. C. Financial budgets. D. Time factor. E. Medical schools and health training institutions. 29. The process of day-to-day follow-up activities to ensure that health programs are proceeding as planned is: A. Evaluation. B. Surveillance. C. Monitoring. D. Implementation. E. Prioritization 30. The best indicator for the assessment of the nutritional status of under-fives children is ... A. Weight for age. B. Height for age. C. Weight for height. D. Head circumference. E. Skinfold thickness 31. Which one is the best indicator for the status of iron storage in the body? A. Hemoglobin concentration. B. Total iron binding capacity. C. Serum iron. D. Serum ferritin. E. Mean corpuscular hemoglobin concentration 32. Learning can be affected by the following conditions, EXCEPT: A. Gender. B. Intelligence. C. Age. D. Motivation. E. Mental health. 33. Which of the following factors tends to increase patient's compliance? A. The presence of a psychiatric disorder. B. The need for long-term treatment. C. Significant behavioral change required. D. Medicines with few side effects. E. Treatment with lower costs.
  • 6. [2007 arab board for community medicine exam paper one (70%)- mcqs.] Dr.AbdulQawi Almohamadi Page 6 of 107 Q 34-35: An investigator wished to determine whether families with a child with attention deficit disorder (ADD) have other family members who develop depression more than do families without children with ADD. Group I was made up of families with a child identified as having ADD. through a community psychiatry clinic. Group II was made up of families also seen at the community psychiatry clinic for diagnoses other than ADD. The groups were followed over one year and the incidence of depressive episodes in the two groups was almost similar. 34. What type of study was this? A. Prospective cohort. B. Case-control. C. Clinical trial. D. Historical cohort. E. Exposure study. 35. The measure of association in this case... A. Requires obtaining also the prevalence rates of ADD in the population. B. Can be measured even if one does not know how large is the population from which participating families were selected. C. May be affected by the recall bias. D. Is the odds-ratio. E. Indicates that ADD is one family member is a protective rather than a risk factor for depression. 36. The sensitivity rate of a test is defined as... A. True negative rate B. True positive rate C. True positives + True negatives D. Hundred minus false positive rates E. Accuracy rate 37. Cross-Sectional Studies ... A. Measure the incidence rate of a disease. B. Measure the prevalence rate of disease. C. Are useful for investigating rare conditions. D. Identify the risk of occurrence of disease associated with specific characteristics. E. Are useful in investigating disease etiology. 38. Which ONE of the following diseases has the highest case- fatality rate? A. Rabies. B. AIDS C. Leprosy D. Hemorrhagic fevers E. Tuberculosis 39. Aluminum sulfate is commonly utilized in... A. Milk purification B. Decreasing air pollution C. Prevention of radiation hazard D. Sewage disposal E. Water purification 40. Solid waste disposal in short-term refugee camps is best done by... A. Land filling of shallow trenches. B. Composting. C. Throwing on the road side. D. Incinerating. E. All of the above. 41. Which one of the following pesticides can cause pulmonary fibrosis? A. Organophosphates. B. Parathion. C. Halogenated hydrocarbons. D. Paraquat. E. Mercury compounds. 42. An important indicator of environmental air pollution is A. Chlorine. B. Fluoride. C. Sulphur dioxide. D. Carbon monoxide. E. Hydrogen. 43. The best method for treating water from a very turbid source is A. Alum coagulation B. Rapid sand filtration C. Activated charcoal D. Ozonation E. Aeration 44. In a hospital-based case-control study of the association between coffee consumption and the occurrence of pancreatic cancer, a group of patients hospitalized after diagnosis of pancreatic cancer was compared to a control group hospitalized for other reasons. The patients hospitalized for pancreatic cancer (cases) were found to consume significantly more coffee than the controls. All of the following statements represent possible explanations for the observed positive association between coffee consumption and pancreatic cancer, EXCEPT: A. Heavy coffee drinkers may also be heavy smokers, so smoking rather than coffee consumption is the relevant causal factor. B. Excessive coffee consumption causes pancreatic cancer. C. The hospitalized controls consumed less coffee, on the average, than individuals in the general population with no history of pancreatic cancer resulting in a spurious association between coffee consumption and pancreatic cancer. D. The cases restricted their coffee intake after being diagnosed with cancer. E. The preferred measure of association in this case is the odds-ratio.
  • 7. [2007 arab board for community medicine exam paper one (70%)- mcqs.] Dr.AbdulQawi Almohamadi Page 7 of 107 45. All the following statements about observational studies are true, EXCEPT: A. Events are observed as they occur in nature, with no active intervention on the part of the investigator. B. The comparison groups may differ with respect to factors related to the response variable. C. Subjects may be followed forward in time from exposure to occurrence of outcome, backward in time from outcome to exposure, or evaluated at a single point in time to concurrently assess outcome and exposure. D. They are especially useful in situations where it is impossible, impractical, or unethical to manipulate exposure to a suspected risk factor. E. None of the above. 46. A pharmaceutical company wishes to evaluate the effectiveness of an antihypertensive agent. It will be submitted for regulatory approval only if it can be shown to lower average systolic blood pressure by at least 10 mmHg compared to a standard drug. The investigators calculate that if they administer the new drug and the standard drug to two groups of 100 subjects each, the study will be 90% certain of detecting the relevant 10 mmHg difference, if it actually exists. The power of the proposed study is A. Greater than the power of a study that included 50 subjects in each treatment group. B. Is defined as the probability of detecting the 10 mmHg difference in systolic blood pressure between the comparison groups if no true difference exists. C. Is equal to 0.90. D. A & C E. B & C 47. A Prospective Cohort Study ... A. Employs subjects known at the onset to have the disease in question. B. Employs subjects known at the onset to be disease- free. C. Employs subjects whose exposure to a suspected risk factor is comparable to that of the control group at the onset of the study. D. Is preferred in the study of rare diseases. E. Is more ethical than a case-control study. 48. Vital statistics in most countries include data on all the following, EXCEPT: A. Death. B. Birth. C. Marriage. D. Divorce. E. Disability 49. The International Classification of Diseases (ICD) is A. A relatively recent concept. B. Needed to compare disease patterns between countries. C. Developed specifically for each of the WHO regional offices. D. Established at the Center for Disease Control in Atlanta (USA). E. Not available for ill-defined signs and symptoms often encountered in primary health care practice. 50. The sequential temporality means that A. That there are more than one cause acting in a specific order to cause disease. B. The relation between cause and effect is for limited time. C. There is an association between cause and effect only in specific seasons. D. The cause has occurred after the effect. E. None of the above. 51. Biological plausibility means that... A. The same findings may be observed by different persons in different places. B. The association between cause and effect is biologically possible. C. Greater levels of exposure lead to greater effects. D. The effect of similar biological factors should be considered. E. None of the above. 52. Which of the following statements is correct regarding population - based health management? A. Epidemiology can make a significant contribution to generating population-based health management data. B. This type of health management focuses on assessing the health status of targeted populations C. This management assesses the financial resources to cover the health needs of a population. D. A & B. E. A & B & C 53. The attributable risk is defined as the... A. Incidence of a disease in the exposed group. B. Incidence of a disease in the non-exposed group. C. Incidence in the total population. D. Incidence in the exposed group divided by incidence in the non- exposed group. E. Excess risk due to the presence of a risk factor in a given population.
  • 8. [2007 arab board for community medicine exam paper one (70%)- mcqs.] Dr.AbdulQawi Almohamadi Page 8 of 107 54. Which value represents the strongest significant relative risk? A. 1.3 (0.8 - 2.1) B. 0.5 (0.4 - 0.6) C. 2.4 (1.2 - 3.6) D. 2.2 (1.2 - 3.6) E. 2.6 (0.7 - 4.5) 55. What is/are the most specific problem(s) associated with survey interviews, compared to postal surveys? A. Higher costs. B. Lower response rates. C. Lower validity. D. More important potential selection bias. E. All of the above 56. In a case of streptococcal sore throat with at least 4 symptoms, what is the probability of obtaining a positive throat culture? A. 2.5% B. 6-7% C. 14.1-16.9% D. 20-55% E. None of the above 57. All of the following disorders are believed to be solely the result of genetic factors EXCEPT: A. Epilepsy. B. Hemophilia. C. Fibrocystic disease of the pancreas. D. Multiple polyposis of the colon. E. Xeroderma pigmentosum. 58. Treatment of a person with AIDS using AZT is what level of prevention? A. Primary prevention. B. Secondary prevention. C. Tertiary prevention. D. Primordial prevention. E. None of the above. 59. Virulence is defined as... A. The propensity of Neisseria meningitides to produce severe and fatal disease. B. The capacity of the HIV virus to survive in defined environments outside the human body. C. The capacity of the influenza virus to enter, survive and multiply inside the host. D. The capacity of toxigenic E.coli to produce the functional, morphologic and pathologic changes that cause symptomatic disease. E. The capacity of Salmonella species to resist common antibiotics. 60. Pathogenicity is defined as... A. The propensity of Neisseria meningitides to produce severe and fatal disease. B. The capacity of the HIV virus to survive in defined environments outside the human body. C. The capacity of the influenza virus to enter, survive and multiply inside the host. D. The capacity of toxigenic E.coli to produce the functional, morphologic and pathologic changes that cause symptomatic disease. E. The capacity of Salmonella species to resist common antibiotics. 61. Fertility refers to A. The capacity to bear children. B. The probability of conceiving in a given month. C. The actual birth of living offspring. D. The probability of conceiving in a given year per 1,000 women. E. The ratio of live births to total pregnancies. 62. All the following techniques of early detection of asymptomatic cancer are considered to be justified, EXCEPT: A. Mammography. B. Breast self-examination in younger women. C. Papanicolaou (Pap) smear. D. Fecal occult blood testing. E. Chest radiography to detect lung cancer. 63. Town B has a higher crude mortality rate as a result of cancer in men than in Town A. However, the age-standardized mortality rate from cancer is higher in Town A than in town B. What conclusions can be drawn from these data? A. There were more deaths from lung cancer in Town A than Town B. B. There were more deaths from lung cancer in Town B than in Town A. C. Town A has a younger age distribution than Town B. D. Town B has a younger age distribution than Town A. E. None of the above conclusions may be made with certainty with the given data. 64. The most frequent cancer site in Middle-East women is A. Lung. B. Breast. C. Colorectal. D. Uterus. E. Ovary. 65. The oligo-element believed to have the greatest influence in inhibiting the formation of free radicals which can cause cancer is A. Iron. B. Selenium. C. Copper. D. Zinc. E. Mercury.
  • 9. [2007 arab board for community medicine exam paper one (70%)- mcqs.] Dr.AbdulQawi Almohamadi Page 9 of 107 66. Which of the following statements about child abuse is correct? A. Child abusers are most often unrelated to the child. B. Most child abusers have psychotic or antisocial personalities. C. More than 95% of all serious head injuries of children under the age of one are due to physical abuse. D. Child abuse is rare in most Arab countries and should therefore not be reported to relevant authorities. E. None of the above. 67. The weight of high-school children who engage in competitive sports is compared to that of students who do not. The best way of testing whether differences are statistically significant between the two groups is to use: A. Chi-square test. B. Correlation coefficient C. Unbiased distribution D. Student's t-test E. Multivariate analysis 68. On a hot summer day in a large urban center, the emergency room department reports an increase in admission for asthma in children and young adults but not among patients suffering from chronic bronchitis. The most likely air pollutant responsible for exacerbation of asthma is: A. Co B. Ozone C. N02 D. Particulate matters E. Lead 69. The most important factor for heat-related illness is: A. Obesity B. Age over 65 C. Female gender D. History of prior heat strokes E. Low socio-economic status 70. Which statement regarding acute flaccid paralysis (AFP) is not correct? A. Countries which do not report AFP may not have a good surveillance system for polio B. The approximate AFP incidence rate is known for any given population C. The reporting of AFP is not mandatory D. The main differential diagnosis for AFP is polio E. AFP cases that are not caused by polio are most often cases of Guillain-Barre syndrome 71. Diseases whose spread is linked to utilization of ventilation systems in big building is: A. Lyme's disease B. Ebola fever C. Asthma D. Rubella E. Legionellosis 72. The target population for early detection of chronic conditions by periodic health examinations or mass screening programs, consists of: A. Asymptomatic, apparently well people B. Cases diagnosed at the primary health care level C. Attendants of out-patient facilities D. Hospitalized in-patients E. Persons who have signs and symptoms of the disease 73. The epidemiological transition means that: A. Chronic diseases are now a thing of the past B. Nutritional problems are not a major public health issue any more C. This generation is expected to survive longer than its grand-parents D. Infant mortality rates have been decreasing steadily E. Infectious diseases are better prevented through immunization than through personal hygiene 74. If rapidly progressive cancers are missed by a screening test, which type of bias will occur? A. Lead-time bias B. Length bias C. Selection bias D. Surveillance bias E. Information bias 75. "The following figure shows that the height of a son is highly correlated (r=l.02) with the height of his father. It indicates a positive linear relationship between the two heights." Which of the following statements is correct? A. The association is not linear B. The correlation is not positive C. The correlation coefficient (r) should not be used in this case D. The value of the correlation coefficient (r ) is not correct E. The height of the son is not correlated with the height of the father
  • 10. [2007 arab board for community medicine exam paper one (70%)- mcqs.] Dr.AbdulQawi Almohamadi Page 10 of 107 76. Electromagnetic fields generated by high-voltage electric power lines have been incriminated in causing A. Lymphoma B. Breast cancers C. Diarrhea D. Emphysema E. Childhood leukemia 77. The fact that the combined effects of several exposures is greater than the sum of the individual effects is known as: A. Latency B. Synergism C. Threshold effect D. Dose-response effect E. Pedestalization 78. Which form of payment encourages physicians to require more procedures and services than may be needed? A. Fees paid by a Health Maintenance Organization capitation system B. A Public Health System in which doctors receive a monthly basic salary C. Fee-for-service without compensation to the patient D. Fee-for-service with third-party repayment to the patient E. None of the above 79. Global warming may be associated with the emergence or re emerging of all EXCEPT: A. Dengue B. Yellow fever C. Rift Valley fever D. Malaria E. Bovine spongiform encephalopathy 80. Knowing that the sample mean is 23, the standard deviation is 0.5 and the sample size is 100. The 95% confidence interval of the mean would approximately be: A. [21-25] B. [22-24] C. [21-24] D. [22.5-23.5] E. [22.95-23.05] 81. Travelers to endemic malaria area should: A. Get a prophylactic vaccine one week before departure and a booster within three weeks of returning B. Consult a doctor for drug dosages to be used while on that trip C. Rely on mosquito repellent and bed screens to prevent bites D. Avoid having sex with someone living in that area E. Remain under medical surveillance for a year if infection with Plasmodium falciparum is suspected 82. In a study of heart surgery, one issue was the reduction effect of a drug on the pulse rate of patients during surgery. The available subjects were divided at random into two groups. (group1) received the drug, the other (group 2) a placebo. What would be the null and the alternative hypothesis statements? A. Ho: Mean1=Mean 2, and H1: Mean 1>Mean 2. B. Ho: Mean1=Mean 2, and H1: Mean 1<Mean 2. C. Ho: Mean1=Mean 2, and H1: Mean 1≠Mean 2. D. Ho: Mean1<Mean 2, and H1: Mean 1=Mean 2. E. Ho: Mean1>Mean 2, and H1: Mean 1=Mean 2. 83. In the control and prevention of meningococcal meningitis: A. The carrier state is common in epidemics B. Man and monkey are reservoirs of infection C. Isolation of patients is of significance in disease control D. The infectious agent is Neisseria meningitides E. No specific immunity follows clinical infection 84. The width of a confidence internal will decrease when which of the following changes occur? A. The desired level of confidence increases B. The subject-to-subject variation in values of the response variable increases C. The sample size increases D. The precision of the estimate of the population mean decreases E. The distance between the sample mean and the population mean increases 85. All of the following infectious diseases are described as zoonosis, EXCEPT: A. Typhoid fever B. Leptospirosis C. Brucellosis D. Rabies E. Salmonellosis
  • 11. [2007 arab board for community medicine exam paper one (70%)- mcqs.] Dr.AbdulQawi Almohamadi Page 11 of 107 86. Which of the following is the most common site of hospital- acquired/ infections A. Surgical wound B. Respiratory tract C. Skin D. Urinary tract E. Gastrointestinal tract 87. All of the following statements about tuberculosis are true, EXCEPT: A. The risk of developing tuberculosis is greater within the first year following infection B. Most cases of tuberculosis occur as a result primary infection C. Routine screening of general populations is no longer recommended D. INH chemoprophylaxis may be given to selected high risk patients over 35 years of age E. Most cases of tuberculosis can be successfully treated with a 6-month drug regimen 88. The incidence of cholelithiasis is increased in all the following, EXCEPT: A. Persons with diabetes B. Persons with chronic hemolytic anemia C. Persons with hypercholesterolemia D. Persons who are obese E. Women 89. Infants of diabetic mother are not at risk of A. Polycythemia B. Hypomagnesaemia C. Neonatal jaundice D. Hypoglycemia E. Traumatic delivery 90. Exposure to dimethyl mercury (organic mercury) found in medical laboratories may cause all the following EXCEPT: A. Cerebellar degeneration B. Paresthesias C. Gingivitis D. Coma E. Visual deficits 91. If the output of a program meets the expected outcome, then the program is described as: A. Adequate B. Productive C. Efficient D. Effective E. Relevant 92. Break point of chlorination means: A. The start the chlorination process B. Free residual chlorine starts appearing. C. The end of chlorination process D. Partial saturation of water with chlorine. E. All of the above 93. Diagnostic criteria for the metabolic syndrome according to the Adult Treatment Panel (ATPIII) include all the following EXCEPT: A. Abdominal obesity B. Dyslipidemia C. High blood pressure D. High fasting glucose E. Microalbuminuria 94. The best food source of iron is: A. Dates B. Whole wheat bread C. Egg yolk D. Raisins E. Milk 95. Health equity is best measured by: A. Infant Mortality Rates B. Child growth C. Survival rates D. Medical rates E. Primary health care (PHC) facilities 96. High parity may be associated with the following EXCEPT: A. Hypertensive disorders of pregnancy B. Cancer of the cervix C. Breast cancer D. A high risk of infant mortality E. Gestational diabetes mellitus 97. Heat stability is lowest for: A. Diphtheria vaccine B. Hepatitis B vaccine C. Dried measles vaccine D. Dried BCG E. Sabin vaccine for poliomyelitis 98. The mental illness is most likely to occur in young adults (15- 24) is: A. Autism B. Schizophrenia C. Affective disorders D. lnvolutional melancholia E. Agitated depression 99. Which statement concerning oral rehydration solution (ORS) is correct? A. Contains salt, sugar and trace of vitamins and minerals B. Salt is included to improve the function of the small intestine mucosa and increase water and sodium absorption C. Will help in preventing vomiting of the child D. Mothers should not stop breast-feeding when giving the ORS E. Mothers should not give ORS to their children in all types of diarrhea
  • 12. [2007 arab board for community medicine exam paper one (70%)- mcqs.] Dr.AbdulQawi Almohamadi Page 12 of 107 100. In maternal care: A. The safe motherhood initiative is related to the work by UNISCO to prevent avoidable causes of deaths among mothers. B. The four pillars of the safe motherhood initiative include antenatal care, female nutrition, hospital delivery and family planning. C. Risk status of pregnancy is determined from the history taking during the first visit to the antenatal D. Age at pregnancy and parity of the mother are example of non avoidable risk factors during antenatal E. Pregnant mothers with previous still births and abortions are considered among moderate to low risk group. 101. In school health service, the following are true, EXCEPT: A. School age groups represent an interval of development that needs medical supervision. B. The pattern of health problems in the community will determine the shape of the school health service. C. It is not recommended to have a doctor in each school as part of the school health service. D. Families might be affected by health education given to their children in schools. E. Teachers have a minor role in the planning and implementation of the school health service. 102. Regarding the status of newborns, infants and children in the world, all the following statements are true, EXCEPT: A. Preterm labor and low birth weight are two major causes of neonatal and perinatal mortality in both developing and developed world B. Birth trauma and infection can be counted as the major causes of death in the developing world C. Respiratory distress syndrome is a health problem in newborn that contributes a high percentage in their mortality and morbidity D. Tetanus neonatorum has become a minor cause of death in the developing world E. Infant mortality in Saudi Arabia can be reduced more by the prevention of low birth weight, labor complication and neonatal infections . 103. Natural passive immunity is obtained through... A. Immunization B. Acquired infections C. Administered antibodies D. Collective immunity E. Trans placental antibodies 104. Which infectious agent is associated with high incidence of liver cancer? A. Hepatitis A Virus (HAV) B. Human Papilloma Virus (HPV) C. Helicobacter pylori D. Hepatitis B Virus (HBV) E. Malaria species 105. All the following features of polio may contribute to its eradication, EXCEPT: A. There exists a safe and inexpensive vaccine against polio B. Most cases of polio infection are asymptomatic C. Polio may be attacked during a season of low incidence D. There is public demand for the eradication of this disease E. Natural or induced immunity is life-long 106. Vaccination may fail because of which factor? A. Some vaccines cannot resist cold weather B. Vaccines are used in populations different from those in which they were developed C. Drugs necessary to activate the vaccine were not taken as prescribed D. A vaccine was licensed for use in human populations without proper controlled trials E. Vaccinated individuals are too old to develop antibodies to the vaccine 107. Acute respiratory infections in children A. Are preventable with measles and whooping cough vaccination. B. Are measured with accurate statistics based on advanced surveillance systems worldwide. C. May cause death as a consequence of pneumonia. D. May be transmitted from domestic animals acting as reservoirs, and such animals should not be allowed near children. E. May be equally severe in developing than in developed countries 108. Control of enteric diseases includes all EXCEPT: A. Washing hands after touching a patient B. Using white gowns C. Isolating the utensils of the patients D. Restricting visits E. Using masks when caring for the patient 109. An endemic disease had a steady frequency over time in a community. Recently, however, a new treatment has been developed that prevents death but does not produce recovery from the disease. The following will occur: A. Incidence of the disease will increase B. Incidence of the disease will decrease C. Prevalence of the disease will increase D. Prevalence of the disease will decrease E. Both incidence and prevalence of the disease will decrease
  • 13. [2007 arab board for community medicine exam paper one (70%)- mcqs.] Dr.AbdulQawi Almohamadi Page 13 of 107 110. The "meningitis belt" refers to: A. A special belt worn by African pilgrims during Hajj. B. Abdominal pain accompanying meningitis infection. C. A rush which appears in the mediastinum following vaccination against meningitis infection. D. Geographical area in central Africa where meningitis is endemic. E. The geographical area in Africa where meningitis has been eradicated. 111. Cigarette smoking may cause all the following, EXCEPT: A. Colo-rectal cancer B. Emphysema C. Stroke D. Larynx cancer E. Bladder cancer 112. The occurrence of an illness at a rate above the expected number is called: A. Epidemic B. Endemic C. Sporadic D. Pandemic E. Zoonotic 113. The "healthy worker effect" causes which kind of bias? A. Selection B. Confounding C. Recall D. Social E. Measurement 114. The Health Belief Model (HBM) explains A. The religious beliefs related to health. B. The cognitive variables affecting the desire to engage in health prevention. C. What people think about causes of serious diseases such as cancer. D. Differences in the trust that people put in the healthcare providers. E. The association between demographic variables and health beliefs. 115. What is the maximum quarantine incubation period to allow a person coming from an infected area to enter a country? A. One incubation period B. Double the incubation period C. Thrice the incubation period D. One month E. Two months 116. A man of 63 had been treated for some years for malignant hypertension. While seriously ill with congestive heart failure, he developed acute appendicitis. Appendectomy was carried out successfully, but the heart condition deteriorated further and he died 2 weeks later. The underlying cause of death was: A. Malignant hypertension B. Congestive heart failure C. Cardiac hypertrophy D. Acute appendicitis E. Appendectomy 117. The target population for early detection of chronic conditions either by periodic health examinations or mass screening programs consists of: A. Asymptomatic, apparently well people. B. Individuals with symptoms and/ or signs of the disease. C. Cases diagnosed at the primary health care level D. Hospitalized in-patients. E. Users of emergency rooms 118. All the following statements regarding smoking are true, EXCEPT: A. As role-models, physicians should abstain from smoking in public B. Passive smoking is a serious health hazard for those sharing living quarters with a smoker C. It is not acceptable that the majority of male medical students would be smokers D. Proper enforcement of rules and regulations should be considered in any smoking control program E. Nicotine addiction is a desirable way of coping with stress 119. Regarding the Gulf War Syndrome: A. It has become a major public health problem in Kuwait after Liberation. B. It is a genetically determined syndrome. C. It is a clearly defined bacteriological disease. D. It is the label given by the UN to the malnutrition and high mortality rates experienced by Iraqi infants and children after the imposition of sanctions. E. It is a collection of ill-defined health problems experienced by soldiers from the coalition who had received a variety of vaccinations prior to serving in the Middle-East during the Kuwait Liberation War 120. Dermatitis, diarrhea and dementia are signs associated with deficiency in vitamin... A. A B. B1 (Thiamine) C. C D. D E. B3 (Niacin) END OF THE EXAM
  • 14. MCQs 2008 Arab Board For Community Medicine Exam Paper 1 (70%) Dr.AbdulQawi Almohamadi Page 14 of 107 MCQs 2008 1. For a man aged 40 to 50 years with no known medical problems, which of the following screening tests should routinely be performed? A. Urine analysis B. Sigmoidoscopy C. Cholesterol D. Prostate specific antigen E. Electroencephalogram 2. Electromagnetic fields caused by high voltage electrical lines have been incriminated in causing... A. Lymphoma B. Breast cancer C. Diarrhea D. Emphysema E. Childhood leukemia 3. The medical records of 73 people with hepatitis B were reviewed to determine if there is an association between their previous vaccination status and their chances of contracting hepatitis B. This type of study is A. A case-control B. A cohort C. Cross-sectional D. Correlational E. Experimental 4. The crude birth rate, the most fundamental fertility measure, uses A. All births in the numerator and total population in the denominator. B. All births in the numerator and women 15-44 years of age in the denominator. C. The sum of all the age-specific fertility rates by single years of age. D. Birth to women in a specific age range in the numerator. E. Total fertility rate minus crude death rate. 5. In an epidemic investigation of an outbreak of unknown origin, which activity should be performed first? A. Confirmation of the diagnosis for cases B. Verification that an epidemic really exists C. Development and testing of an explanatory hypothesis D. Proposal of measures for control of the health problem E. Search for additional cases of the suspect disease 6. When a screening for HIV is performed, the ELISA test is repeated if positive. If the second test is positive, a Western blot is performed for confirmation. This serial interpretation has which of the following effects? A. Increases both the sensitivity and specificity B. Increases the predictive positive value and slightly decreases the sensitivity C. Increases the predictive positive value and slightly increases the sensitivity D. Increases both the specificity and the predictive negative value E. Increases the sensitivity only 7. The two most important values usually necessary as a description of the distribution of a series of quantitative observations are... A. Standard deviation and mean B. Median and variance C. Mode and range D. Range and mean E. Size of sample and standard deviation 8. The proportion of death cases from a specific disease divided by all deaths is called... A. An incidence rates B. A point prevalence C. A case fatality D. A proportional mortality rates E. A disease-specific mortality rate 9. Which of the following risk factors is associated with the greatest number of cancer deaths? A. Diet B. Alcohol C. Smoking D. Obesity E. Environmental exposure 10. A 21-year-old college student presented to the student health center with complaints of cough and fever for a few days. An erythematous maculopapular rash and Koplick's mucosal spots are found on physical examination. Which of the following statements is true concerning this illness? A. This illness is more common and more severe in children compared with infants or adults B. In the typical form, the rash is not a common symptom C. Conjunctivitis, excessive lacrimation and photophobia are common symptoms D. Prompt administration of immune globulins soon after exposure does not alter the course of the illness E. Antibody protection after the infection lasts for only 2 to 3 years
  • 15. MCQs 2008 Arab Board For Community Medicine Exam Paper 1 (70%) Dr.AbdulQawi Almohamadi Page 15 of 107 Q11-12. : A 72-year-old man has a 1-month history of angina pectoris occurring at rest, lasting up to 20 minutes. Episodes have been increasing in frequency and intensity over the past few weeks. He has smoked 2 packs of cigarettes per day for 50 years and has been under treatment for hypertension for the last 15 years. 11. Which type of angina best describes this patient's pain? A. Chronic stable angina B. Variant angina (Prinzmetal's) C. Silent ischemia D. Unstable angina E. Traditional angina 12. The initial step in the management of patients should be: A. Admission to the hospital for further work-up B. Observation for several hours in the emergency room until pain resolves C. Obtain ECG and follow patient on outpatient basis if ECG is normal D. Place patient on aspirin 325 mg per day and instruct him to return to an office follow-up visits in two E. Reassure patient and send him home without further follow- up 13. Which of the following is NOT true about the prevention and treatment of Haemophilis influenzae infection? A. Chemoprophylaxis with isoniazid for household contacts under the age of 6 has been recommended in cases of H.influenzae meningitis B. For normal healthy infants, H.influenzae immunization begins at 2 months of age C. A three-stage vaccination is available for primary immunization and a booster D. Treatment for life threatening illness from the H.influenza may include both chloramphenicol and ampicillin. E. Protection with the vaccine lasts 5 to 10 years or longer. 14. The following are primary preventive measures of diabetes EXCEPT: A. Genetic counseling B. Encouraging exercise C. Controlling weight D. Giving appropriate doses of insulin E. Encouraging complete immunization against infection 15. Precautions in the administration of oral polio vaccine include avoidance of vaccination in the presence of which of the following conditions? A. An Immunodeficient household contact B. Prematurity C. Diarrhea D. Breast feeding E. Low-grade fever 16. A pregnant 23-year-old women comes into the primary care center with a diagnosis of pelvic inflammatory disease. She is at LEAST risk for developing which of the following A. Premature rupture of membranes B. Spontaneous abortion C. Chorioamnionitis D. Prematurity E. Low birth weight infant 17. A 36-year-old obese man with documented diastolic hypertension presents to your office. Which of the following is LEAST likely to help improve his diastolic hypertension? A. Decreasing sodium intake B. Weight loss program C. Avoidance of excessive alcohol intake D. Decreasing dietary fat intake E. Regular exercise program 18. Human papilloma virus (HPV) infection has been strongly associated with all the following cancers EXCEPT: A. Vulvar B. Testicular C. Anal D. Cervical E. Vaginal 19. The following drugs are effective treatments of Salmonella typhi EXCEPT: A. Chloramphenicol B. Trimethoprim-sulfamethoxazole (Septrim) C. Ampicillin D. Ketoconazole E. Ciprofloxacin 20. Which of the following is true regarding dengue fever? A. The infectious agent involved in dengue fever is a parasite similar to Plasmodium species B. Fatalities are common with this disease even in the absence of hemorrhagic fever C. The mode of transmission is by the bite of infected sand flies D. Prevention depends on the use of vaccine and post-exposure immune globulin E. Onset is sudden and violent
  • 16. MCQs 2008 Arab Board For Community Medicine Exam Paper 1 (70%) Dr.AbdulQawi Almohamadi Page 16 of 107 21. All of the following are associated with a higher mortality risk from diseases EXCEPT: A. Having a low socio-economic B. Status being female C. Having few friends D. Moving from one place to the other frequently E. Being divorced 22. Which agent is most likely responsible for a chronic infection leading to cirrhosis or primary hepatocellular carcinoma? A. HAV B. HBV C. HCV D. HDV E. HIV 23. The administration of human immune globulin after exposure to the Hepatitis B virus is an example of… A. Secondary prevention B. Health promotion C. Passive immunity D. Hypersensitivity E. Cross reactivity 24. All the following have been identified as major risk factors for Coronary Heart Disease, EXCEPT: A. High serum cholesterol B. Uncontrolled elevation in blood pressure C. Cigarette smoking D. Lack of daily vigorous exercise E. Family history 25. Which of the following vaccines would most likely be dangerous to a person with immunodeficiency? A. Tetanus B. Typhoid C. Diphtheria D. HBV E. Measles 26. Routine antenatal care for normal pregnant women should include all the following EXCEPT: A. Weight checks B. Blood pressure checks C. Hemoglobin level tests D. Urine examination E. Electronic fetal monitoring 27. Of the following, the most effective method of family planning is... A. Contraceptive pills B. Withdrawal C. Safe period D. Condom E. Traditional methods 28. The value of alpha serves as protection against... A. Type I error B. Type II error C. Type III error D. False negative results E. Inadequate sample size Q 29 – 31 : A count of cases found the following distribution of observation: 1, 2, 2, 2, 3, 5, 5, 6,6,18. 29. The mean is : A. 3 B. 4 C. 5 D. 6 E. 7 30. The median is : A. 3 B. 4 C. 5 D. 6 E. 7 31. The mode is: A. 6 B. 5 C. 4 D. 3 E. 2 32. A randomized clinical trial was designed to compare two different treatment approaches for epileptic attacks. The purpose of randomization in this study was to A. Obtain treatment groups of similar size B. Select a representative sample of patients for study C. Increase patient compliance with treatment D. Decrease the likelihood that observed differences in clinical outcome are due to chance E. Obtain treatment groups with comparable baseline values 33. The purpose of the informed consent in a clinical trial is to A. Increase the patient's knowledge of possible risks and benefits of treatment options B. Increase the level of patient's participation C. Minimize the effect of placebo D. Decrease the likelihood of patient blinding to treatment assignment E. Prevent the occurrence of malpractice suits
  • 17. MCQs 2008 Arab Board For Community Medicine Exam Paper 1 (70%) Dr.AbdulQawi Almohamadi Page 17 of 107 34. Breast feeding usually improves all the following EXCEPT: A. Uterine involution B. Bonding C. Infant nutrition D. Infant immune system E. Prevention of HIV infection 35. Of the following environmental hazards, which one is the only exposure with a Dose-response effect? A. Ionizing radiation B. Noise C. Heat D. Lead E. Carbon monoxide 36. The site of most frequent and serious contact with environmental hazards is A. Skin B. Hands C. Eyes D. Gastrointestinal Tract E. Lungs Q37- 38: Dr.Baha observed a rapid recovery in one of his Hepatitis C patients who sat in a hospital solarium daily. A review of the literature revealed no reported effects of the sun on the convalescence of hepatitis patients, so Dr Baha published a summary of this patients unusual recovery. 37. Which of the following study designs has Dr.Baha used? A. Case report B. Case-control C. Environmental cohort D. Clinical trial E. Cross-sectional report 38. Dr Baha enlists the support of several gastroenterologists and designs a study whereby Hepatitis C patients, upon hospital admission are alternately assigned to receive either 2 hours of sun exposure daily or no sun exposure. Which of the following study designs has Dr Baha used? A. Case report B. Case-control C. Environmental cohort D. Clinical trial E. Cross-sectional report 39. An indicator measuring short term outputs in Extended Programs of Immunization (EPI) is A. Number of refrigerators available in centers to keep vaccines in the cold chain B. Number of vaccination sessions C. Coverage rates D. Decreases incidence of measles E. Number of trained health workers in EPI 40. The leading cause of fatal home injuries among children under five is A. Burning B. Explosions C. Poisoning D. Falls E. Drowning 41. The best method for treating water from a very turbid source is ... A. Alum coagulation B. Rapid sand filtration C. Activated charcoal D. Ozonation. E. Aeration 42. The pollutants globally known as Volatile Organic Chemicals tend to concentrate in: A. Stagnant water B. Air C. Fatty tissues D. Edible fish tissue E. Soil 43. All of the following cancers have been associated with occupational exposure EXCEPT: A. Bladder cancer B. lung cancer C. Hematopoietic cancer D. Breast cancer E. Liver cancer 44. Principals of Primary Health Care include all the following items EXCEPT: A. Multisectorial approach to health B. Appropriate technology C. Equity of resource D. Distribution sanitation E. Emphasis on prevention 45. Which one of the following is a true statement regarding the "need" for medical care and the "demand" for medical care? A. They are different because patients need care but physicians demand it B. They are different because "need" is professionally defined and "demand" is patient generated C. Need" usually surpasses "demand" D. They are different because "need" takes cost into consideration E. They are the same when barriers to care are minimal.
  • 18. MCQs 2008 Arab Board For Community Medicine Exam Paper 1 (70%) Dr.AbdulQawi Almohamadi Page 18 of 107 46. Medical ethics are founded on all of the following basic principles, EXCEPT: A. Disclosure of information B. Non-malfeasance C. Beneficence. D. Justice E. Respect for patient's autonomy 47. Which statement characterizes families in which spouse abuse occurs? A. Husbands of battered women have higher education than the ones who are not B. Women who were abused as children are more likely to be abused by their spouse C. Women whose mothers suffered abuse are more likely to be abused than those who did not witness D. In the Arab world, men who batter women should expect to be prosecuted E. Older women are more at risk of spouse abuse than younger ones 48. Emotional disturbance of the post-partum A. Are more common after the birth of a male child B. Are more common with increasing parity C. Rarely last for more than 1 week D. Occur in at least 20%of women E. Can be predicted through psychometric tests in pre-natal care 49. When debating whether providing or withholding medical treatment is ethical, the LEAST important consideration should be... A. The indications for medical intervention B. Expected quality of life after the treatment C. Patient's preferences D. Physician's preferences E. Economic factors 50. The nutritional disease characterized by depression, dermatitis and diarrhea is... A. Ariboflavinosis B. Beriberi C. Scurvy D. Pellagra E. Kwashiorkor 51. Which of the following statements concerning iron requirements is correct? A. Requirements of women are not affected by menopause B. Requirements are the same in men and women C. They increase during pregnancy D. Can be satisfied by cow's milk E. Iron requirements during pregnancy does not vary by the pregnancy's trimester 52. In a double-blinded clinical trial concerning the treatment of osteoarthritis, half of the patients received a new drug and the other half received a pharmacologically inert substance. Two thirds of the patients in the former group and one third of the patients in the latter group reported relief of symptoms. The patients' perceptions of improvement on treatment with an inert substance is best described as... A. Intention to treat B. Confounding effect C. Placebo effect D. Type II error E. False positive results 53. No difference in the 5-year survival percentages is observed in one small clinical trial, despite the fact that the new treatment is very promising. Failure to detect a benefit for the new treatment is best described as... A. Observer bias B. Placebo effect C. Type I error D. Type II error E. Survivor bias 54. Routine chest X-rays will result in the early diagnosis of a lung cancer case, with no impact on delaying the eventual time of death. The longer apparent duration of such a case can be attributed to the A. High sensitivity of chest X-rays B. High correlation C. Lead-time bias D. Length bias E. Low prevalence of lung cancer Q55-56: During an 8-hour shift in a building, 30 employees (20 females and 10 males) visited the company physician with complaints of nausea, vomiting, headache and dizziness. All affected individuals responded to supportive treatment and were sent home. 55. If 600 persons worked in the building, then the attack rate was... A. 3% B. 5% C. 10% D. 20% E. 30% 56. If 400 females and 200 males worked in the building, the male to female risk ratio was... A. 0.3 B. 0.5 C. 1 D. 2 E. 3
  • 19. MCQs 2008 Arab Board For Community Medicine Exam Paper 1 (70%) Dr.AbdulQawi Almohamadi Page 19 of 107 Q57-59.: A survey of all workers was conducted to determine whether other persons were affected beyond those who sought care at the physician's office. All 600 employees were surveyed, and 400 questionnaires were completed and returned. A total of 80 persons reported symptoms consistent with the syndrome observed among workers who sought medical attention. 57. The response rate to the questionnaire is nearest to... A. 5.0% B. 7.5% C. 20.0% D. 37.5% E. 67.0% 58. Based on the survey data, the attack rate was... A. 5.0% B. 7.5% C. 20.0% D. 37.5% E. 67.0% 59. In this case, the possibility of a secondary attack among contacts would most likely indicate... A. A point exposure B. Vector transmission C. Sexual transmission D. Fecal-oral contamination E. Person-to-person transmission 60. Which of the following is the best measure to estimate the proportion of children newly entering school with auditory defects? A. Incidence rate B. Prevalence C. Risk ratio D. Median survival E. Case fatality 61. In a cohort study of hormone replacement therapy (HRT) and risk of coronary artery disease (CAD), high-income level is associated with both HRT and risk of CAD. Which statement best describes the effect of "high income" as a variable in this analysis? A. Selection bias B. Misclassification bias C. Confounding D. Ecological fallacy E. Random error 62. In a cohort study of the relationship between dietary supplementation and the occurrence of hip fractures, the relative risk of hip fractures with supplementation was 0.5 with a 95% CI of 0.18 - 1.41. The correct interpretation of this finding is that… A. A significant association exists between calcium supplementation and lower risk of hip fracture. B. The association between calcium supplementation and risk of hip fracture should not have been measured with the relative risk. C. The risk of hip fracture with calcium supplementation is not statistically significant from the risk. D. One can conclude with 95% confidence that calcium supplement protects against hip fracture. E. A Chi-square test should be performed to assess the statistical significance of the association. Q63-66. : A sea resort has a population of 99,000 which can be divided into three age ranges: 25 - 44, 45-64 and 65 and older. Each comprises one third of the population. In July 2003, 100 cases of hepatitis A occurred in the resort and were traced to contaminated oysters. Of these 100 cases, 20 were between ages 25-44, 10 between ages 45-64 and five over the age of 64 ultimately proved fatal. Before 2003 no cases had been reported among the inhabitants of the resort. 63. What is the crude death rate (per 1000 residents) in the resort? A. 0.354 B. 1.01 C. 1.06 D. 35.4 E. Cannot be determined from the given data 64. What is the incidence of hepatitis A (per 1000) in July 2003? A. 0.35 B. 1.01 C. 40 D. 350 E. Cannot be determined from the given data 65. What is the age specific mortality rate (per 10,000) for residents over 64 years of age? A. 1.52 B. 3.03 C. 3.54 D. 4.55 E. 6.06 66. What is the case fatality rate for hepatitis A? A. 3.03 per 10,000 B. 3.54 per 10,000 C. 350 per 1000 D. per 1000 E. Cannot be determined from the given data
  • 20. MCQs 2008 Arab Board For Community Medicine Exam Paper 1 (70%) Dr.AbdulQawi Almohamadi Page 20 of 107 67. Regarding statistical tests, which of the following is NOT true? A. Paired t-test is useful in the analysis of qualitative data B. The Student's t-test is used to compare two independent sample means with an assumption of equal C. The z-score is the number of standard deviations at which a value in a normally distributed population lies away from the mean D. Chi-square test is used in testing the relationship between two proportions E. The correlation coefficient is used to test the relation between two continuous variables 68. Which of the following changes will decrease the width of a confidence internal? A. Increase in the level of confidence B. Increase in subject-to-subject variation in response variables C. Increase in the sample size D. Decrease in the precision of the estimate of the population mean E. Increase between the sample mean and the population mean Q69-72.: Assume that you are evaluating a new screening test in a population of 1000 people, of whom 75 are known cases for the disorder under screening. Of the 75 afflicted persons, 50 yield positive test results; 75 of the normal people also have positive test results. 69. From this information, the sensitivity for the test is A. 85% B. 67% C. 58% D. 92% E. Cannot be measured with the data provided 70. From this information, the specificity for the test is A. 9% B. 33% C. 40% D. 87% E. 92% 71. From this information, the positive predictive value for the test is A. 60% B. 40% C. 20% D. 10% E. 97% 72. From this information, the negative predictive value for the test is A. 33% B. 40% C. 50% D. 88% E. 97% 73. Which one of the following diseases has not been linked to cigarette smoking? A. Carcinoma of the larynx B. Hypertension C. Abruption of placenta D. Carcinoma of the colon E. Alzheimer's disease 74. Of the following factors listed, which is the most important factor in determining the success of a smoking cessation program in an individual? A. The desire of the patient to quit smoking B. An effective pharmacologic agent C. The inclusion of a behavior modification component to the program D. Physician's advice to quit smoking E. Repeated office visits 75. Which of the following facts regarding colon cancer screening is correct? A. The US Task Force recommends only two screening methods: fecal occult, blood testing and colonoscopy. B. Patients should be informed of the risk of perforation or serious bleeding from colonoscopy is about one/thousand procedures in the community C. Colonoscopy has been proved to reduce mortality in large scale screening trials D. Colon preparation for colonoscopy involves the administration of two enemas before the procedure E. Fecal occult blood testing involves virtually no preparation on the part of the patient 76. Which one of the following is NOT a risk factor for breast carcinoma? A. First degree relative with breast cancer B. Nulliparity C. Birth of first child after age 35 D. Early menarche E. Breast feeding Results Positive Negative Total Disease 50 25 75 Healthy 75 850 925 Total 125 875 1000
  • 21. MCQs 2008 Arab Board For Community Medicine Exam Paper 1 (70%) Dr.AbdulQawi Almohamadi Page 21 of 107 77. Which of the following settings is NOT acceptable for the delivery of bad news? A. Physician's office B. Quiet room in a hospital setting C. Patient's home D. Private room E. Shared hospital room 78. An elevated triglyceride level is associated most closely with which of the following problems? A. Impaired glycemic control B. Hyperthyroidism C. Weight loss D. Very Low Density Lipidemia (VLDL) E. Elevated total serum cholesterol level 79. What is the mode of action of omeprazole? A. H1 receptor angonist B. H2 receptor antagonist C. Proton pump inhibitor D. Cytoprotection E. Anticholinergic action 80. Which of the following statements regarding H. pylori is false? A. The acute infection is self limited and will resolve without antibiotics B. Organisms are found in human feces but not in saliva C. Income and socio-economic factors do not influence prevalence D. Approximately 25% of adults are colonized by the age of 50 years E. Aminoglycosides are indicated as first line of treatment 81. Which of the following is the LEAST common complaint in a victim of spousal abuse? A. Back pain B. Headache C. Dyspareunia D. Spousal abuse itself E. Abdominal pain 82. Blood that is used for transfusion purposes is screened routinely for serologic markers of all the following diseases EXCEPT: A. Hepatitis B virus B. Herpes simplex virus C. Syphilis D. Cytomegalovirus E. HIV 83. Which of the following is the most common site of hospital-acquired infections? A. Surgical wound B. Respiratory tract C. Skin D. Urinary tract E. Gastrointestinal tract 84. All the following are clearly associated with acute episodes of asthma EXCEPT: A. Emotional stress B. Elevated ozone concentrations C. Aspirin D. Caffeinated beverages E. Upper respiratory infection 85. The most important risk factor for developing osteoporosis is A. Smoking B. Alcohol use C. Lack of physical activity D. Age E. Obesity 86. Possible risk factors of the development of colon cancer include all the following EXCEPT: A. Irritable bowel syndrome B. Diet rich in animal fat. C. Inflammatory bowel disease. D. Familial polyposis, E. Uretrosigmoidostomy 87. A person working in hot environment and taking a lot of water but no salt will develop a condition known as heat A. Stroke B. Syncope C. Cramps D. Exhaustion E. Hyperpyrexia 88. Effects of greenhouse gases include all the following EXCEPT: A. Spread of insect born diseases to new areas B. Natural weather disasters C. Summers becoming hotter D. Submersion of low-lying areas E. Ozone depletion 89. Among the problems facing health manpower planning in developing countries are the following EXCEPT: A. Improper mix of different categories of staff B. Low demand for nursing staff C. Migration D. Shortage in numbers E. Faulty training curriculum 90. In postnatal care A. One medical examination is enough B. Breast-feeding promotion is crucial C. It is advisable to continue health supervision until the uterus returns to the pelvis D. Exercise is contraindicated E. Home visits by the health worker are advisable for all mothers
  • 22. MCQs 2008 Arab Board For Community Medicine Exam Paper 1 (70%) Dr.AbdulQawi Almohamadi Page 22 of 107 91. The marginal cost in the context of health care refers to the cost of A. Delivering one extra unit of services B. Training and recruiting staff C. Buildings and equipment D. All daily activities E. Care consumption 92. High parity may be associated with the following EXCEPT: A. Hypertensive disorders of pregnancy B. Cancer of the cervix C. Breast cancer D. High risk of infant mortality E. Gestational diabetes mellitus 93. The best indicator for the status of iron stores in the body is... A. hemoglobin concentration B. Total iron binding capacity C. Serum iron D. Serum ferritin E. Mean corpuscular hemoglobin concentration 94. Which is the process of day to day follow up of activities to ensure that health programs are proceeding as planned? A. Evaluation B. Surveillance C. Monitoring D. Implementation E. Fixing priorities 95. In establishing communication with the patient, it is said that the eyes are the most expressive part of the face. They are the most useful organ for identifying which one of the following A. Anger B. Fear C. Sincerity D. Depression E. Disgust 96. The mental illness most likely to occur in young adults (15-24) is… A. Autism B. Schizophrenia C. Affective psychosis D. Involutional melancholia E. Agitated depression 97. Placing blame for one's own difficulties upon others is considered as... A. Displacement B. Rationalization C. Projection D. Reaction formation E. Obsession 98. Which of the following factors is a major factor in increasing patient's compliance? A. The absence of a psychiatric disorder B. Having a chronic disease requiring long-term treatment C. The need for a significant behavioral change D. Prescribing medicines with few side effects E. Prescribing treatments with lower costs 99. The association between low birth weight and maternal smoking during pregnancy can be studied by obtaining smoking histories from women at the time of their prenatal visit and correlating it with newborn's birth weight. What type of study is this? A. Clinical trial B. Cross-sectional C. Prospective cohort D. Retrospective E. Correlational 100. In order to determine the relationship between serum levels of sodium and antidiuretic hormone (ADH) in patients who have meningitis, which is the most appropriate study design? A. Repeated measurement of sodium and ADH in a patient B. Measurement of sodium and ADH in a set of patients C. Measurement of sodium in a set of patients and of ADH in a different set of patients. D. Measurement of ADH in a set of patients and a set of controls. E. All the above 101. All the following statements regarding the normal (Gaussian) distribution are true EXCEPT: A. The mean = median = mode . B. Approximately 50 percent of observations are greater than the mode. C. Approximately 68 percent of observations falls within1 standard deviation of the mean D. The number of observations between O and1standard deviation from the mean is the same as the number between 1 and 2 standard deviations from the mean. E. The shape of the curve does not depend on the value of the mean 102. Which of the following mechanism could discourage the overutilization of health services? A. Full health care coverage insurance plans B. Deductibles and co-payment C. Government supported health care delivery system D. Home health care plans E. Hospice care
  • 23. MCQs 2008 Arab Board For Community Medicine Exam Paper 1 (70%) Dr.AbdulQawi Almohamadi Page 23 of 107 103. Which one of the following pesticides is capable of causing pulmonary fibrosis? A. Organophosphates B. Parathion C. Halogenated hydrocarbons D. Paraquat E. Organochlorines 104. The object of disinfection of water is to … A. Reduce the bacterial population B. Destroy fecal material C. Prevent bacterial toxin formation D. Limit growth of pathogenic E. Bacteria remove turbidity 105. Which of the following agents are most resistant to disinfection with chlorinated water? A. Hepatitis virus species B. Schistosomes C. Coliform organisms D. Cysts of E. histolytica E. Enteric bacteria 106. Chemical food poisoning associated with preparations added to raw meat to preserve the red color is most commonly due to... A. Antimony B. Arsenic C. Cadmium D. Nicotinic acid E. Nitrates 107. Which of the following general effects would be most characteristic of an ascent to an altitude of 4,000 meters? A. Deterioration of voluntary muscle control B. Impairment of ocular balance C. Impairment of hearing D. Increased night oxygen requirements E. Visual acuity less than 50%of normal 108. Caisson disease is the result of A. Swelling of synovial tissues B. Carbon dioxide saturation in the blood C. Air in the intestinal tract D. Nitrogen bubbles in the blood stream E. Oxygen bubbles in soft tissues. 109. In DOTS, all the following are true EXCEPT: A. This method can solve the problem of defaults in treatment. B. It is a program with a high-cost effectiveness. C. It is designed to be used by trained doctors. D. Can prevent the development of drug resistance. E. Has up to 96% cure rate. 110. The longitudinal research approach is characterized by A. The same group being studied over extended periods of time. B. Different groups being studied over extended periods of time. C. Individuals being studied at different intervals throughout their life cycle. D. Groups of individuals of overlapping ages being tested periodically in similar dimensions. E. Different dimensions of behavior being examined at different stages of development to determine short-range effects of longitudinal analysis. 111. The bacteria that are involved in nosocomial infections are transmitted most often by: A. Airborne matter. B. Fomites. C. Exposure to a common source. D. Indwelling catheters. E. Direct contact via hands. 112. Which one of the following problems is a documented hazard related to the use of oral contraceptives? A. Breast cancer B. Vaginal cancer C. Osteoporosis D. Thromboembolism E. Brain tumor 113. Patients indicate their satisfaction with treatment by responding to a question with four options: very dissatisfied, dissatisfied, satisfied, and very satisfied which type of scale is this? A. Nominal scale B. Ordinal scale C. Internal scale D. Ratio scale E. Categorical scale 114. State the most appropriate statistical test to analyze data comparing the serum cholesterol before and after ingestion of a hamburgers in a sample of persons. A. Student's t-test . B. Multiple regression. C. Paired t-test . D. Correlation study. E. Chi-square
  • 24. MCQs 2008 Arab Board For Community Medicine Exam Paper 1 (70%) Dr.AbdulQawi Almohamadi Page 24 of 107 115. The difference between impairment and disability is that A. Impairment is socially defined. B. Disability is socially defined. C. Disability is medically defined. D. Impairment is not permanent. E. Disability is permanent. 116. The ability to produce a live-born baby is defined as A. Fertility B. Fecundity C. Fertility rate D. Crude birth rate E. Total fertility rate 117. "Perceived susceptibility, severity, obstacles and benefits" are determinants of behavior modification in which theoretical model? A. Health belief model B. Social learning model C. Theory of planned behavior D. Stages of change theory E. Triad model 118. The most consistent parental behavior found in cases of child abuse is... A. Inconsistency in discipline B. Employment of harsh mental punishment C. Rejection D. Parents treating the child as they were treated by their parents E. Correction 119. A worker diagnosed with silicosis is at highest risk of developing: A. Lung cancer B. Tuberculosis C. Emphysema D. Bronchiectasis E. Arteriosclerosis 120. The most frequent cancer site in Middle-East women is A. Lung B. Breast C. Colorectal D. Uterus E. Ovary ************************ GOOD
  • 25. 2014 Exam Paper (70%)- MCQs (Duration 150 minutes) Dr.AbdulQawi Almohamadi Page 25 of 107 MCQs 2014 1. Epidemiology can be defined as the study of A. The etiology of disease in humans B. The determinants of disease frequency in humans C. The distribution and determinants of disease frequency in humans. D. The patterns of health care organization and financing. E. The-distribution of disease in case of an epidemic 2. The association between low birth weight and maternal smoking during pregnancy can be studied by obtaining smoking histories from women at the time of their prenatal visit and then subsequently correlating birth weight with smoking histories. What type of study is this? A. Clinical trial B. Cross-sectional C. Prospective (cohort) D. Retrospective case-control E. None of the above. 3. Which of the following measures is used frequently as a denominator to calculate the rate of a disease? A. Number of cases observed B. Number of new cases observed. C. Number of asymptomatic cases. D. Person-years of observation. E. Persons lost to follow-up. 4. In the study of the cause of a disease, the essential difference between an experimental study and an observational study is that in the experimental investigation A. The study is prospective. B. The study is retrospective. C. The study and control groups are of equal size D. The study and control groups are selected on the basis of history of exposure to the suspected causal factor E. The investigators determine who is and who is not exposed to the suspected causal factors Questions (5 – 6): To determine whether prenatal exposure to tobacco smoke is a cause of undescended testes in newborns, the mothers of 100 newborns with undescended testes and 100 newborns whose TESTES bad descended, were questioned about smoking habits during pregnancy. The study revealed an odds ratio of 2.6 associated with exposure to smoke, with 95% confidence intervals from (1.1 to 5.3) . 5. Which of the following statement is true? A. The odds ratio could be falsely elevated by the inclusion of infants whose testes were descended (but retractile) in the case group (misclassification bias). B. The odds ratio could be falsely elevated by recall bias if parents of affected infants were more likely to remember or report their exposures. C. Because the cases are newborns, but the exposure data came from their mothers, this is not a true case-control study. D. Since the study was not blind, it is impossible to rule out a placebo effect E. None of the above 6. Which of the following statements is true? A. The results provide no evidence that maternal cigarette smoking is associated with undescended testes in the offspring B. If the study results are accurate, they suggest that a baby boy whose mother smoked is about 2.6 times as likely to be born with testes undescended as a baby boy whose mother did not smoke C. The fact that the confidence interval excludes 1 indicates that P>.05 D. The 90 percent confidence interval for these results would probably include 1.0 E. None of the above 7. In order to determine the relationship between serum levels of sodium and antidiuretic hormone (ADH) in patients, who have meningitis, the most appropriate study design would be : A. Repeated measurement of sodium and ADH in a patient B. Measurement of sodium and ADH in a set of patients C. Measurement of sodium in a set of patients and of ADH in a different set of patients. D. Measurement of ADH in a set of patients and a set of controls. E. None of the above. 8. Assume that the annual death rate from lung cancer for British doctors is 160 per 100,000 among heavy smokers compared with 8 per 100,000 among non smokers. What is the relative risk of dying of lung cancer for smokers compared with nonsmokers? A. 152 B. 20 C. 19 D. 8 E. None of the above
  • 26. 2014 Exam Paper (70%)- MCQs (Duration 150 minutes) Dr.AbdulQawi Almohamadi Page 26 of 107 P= c (1 + r)ⁿ Chance =1 - (1- 0.1)³⁰ 9. Randomization is a procedure used for assignment or allocation of subjects to treatment and control groups in experimental studies. Randomization ensures: A. That assignment occurs by chance B. That treatment and control groups are alike in all respects EXCEPT treatment C. That bias in observations is eliminated D. That placebo effects are eliminated E. None of the above 10. In study of the cause of lung cancer, patients who disease were matched with controls by age, sex, place of residence and social class. The frequency of cigarette smoking was then compared in the two groups. What type of study was this? A. Prospective (cohort) B. Retrospective ( case- control) C. Clinical trial D. Historical prospective E. None of the above Questions 11-14; During 1999, 200 newly diagnosed cases of diabetes occurred in city X, which has a population of 10,000 (estimates on July 1, 1999). At the beginning of the year, there were a total of 800 patients with diabetes in the city. During the year, 40 patients died of the complications of diabetes. 11. What was the average annual incidence of diabetes during 1999? A. 4 per 1000 B. 80 per 1000 C. 100 per 1000 D. 200 per 1000 E. 20 per 1000 12. What was the approximate point prevalence of diabetes on January 1, 1999? A. 20 per 1,000 B. 80 per 1,000 C. 200 per 1,000 D. 800 per 1.000 E. 1000 per 1,000 13. What was the prevalence of diabetes on December 31,1999: A. 16 per 1,000 B. 20 per 1,000 C. 80 per 1,000 D. 96 per 1,000 E. 1000 per 1,000 14. What was the mortality due diabetes during 1999? A. 0.4 per 1,000 B. 1.6 per 1,000 C. 2.0 per 1,000 D. 4.0 per 1,000 E. 9.6 per 1,000 15. The sampling method in which each individual of the total group has an equal chance of being selected in the sample is : A. Systematic sampling. B. Paired sampling. C. Simple random sampling. D. Stratified sampling. E. Cluster sampling. 16. A sample of 1,000 people includes 120 who are hearing impaired and 50 who are diabetic. If the number who are both diabetic and hearing impaired is 6, then. A. Diabetes and hearing impairment appear to be independent. B. Diabetics appear to be protected from hearing impairment. C. Diabetics appear to be at greater risk of hearing impairment. D. There is an interaction between diabetes and hearing impairment E. There is not sufficient information to state any of the above. 17. A measures of the amount of variation of a set of values about the mean of the set is the: A. Regression coefficient B. Standard error of the mean C. Standard deviation D. Range E. Correlation coefficient 18. Suppose that each time an individual receives pooled blood products, there is a 10 percent chance of that person developing a cytomegalovirus (CMV) infection. If an individual receives pooled blood products on 30 occasions, what is his or her chance of developing cytomegalovirus. A. 300 percent B. 3 percent C. 30 percent D. 95.8 percent E. 10 percent 19. All the following statements regarding the normal (Gaussian) distribution are true EXCEPT: A. The mean = median =mode B. Approximately 50 percent of observations are greater than mode. C. Approximately 68 percent of observations falls within 1 standard deviation of the mean D. The number of observations between 0 and 1 standard deviation from the mean is the same as the number between 1 and 2 standard deviations from the mean. E. The shape of the curve does not depend on the value of the mean.
  • 27. 2014 Exam Paper (70%)- MCQs (Duration 150 minutes) Dr.AbdulQawi Almohamadi Page 27 of 107 20. Which of the following statements concerning statistical inference is correct? A. If the p value = 0.05 then there is a 95 percent probability that the results didn't occur by chance B. The null Hypothesis generally states that there is a difference between the groups C. If the p value is sufficiently high, the null hypothesis is not rejected D. Knowledge of the sampling method is not important in determining Statistical significance. E. None of the above. 21. The probability of being born with condition A is 0.10 and the probability of being born with condition B is 0.50 If conditions A and B are independent, what is the probability of being born with either condition A or condition B (or both)? A. 0.05 B. 0.40 C. 0.50 D. 0.55 E. 0.60 22. In nine families surveyed, the numbers of children per family were 4, 5, 2, 2, 4, 3, 2, 1, 7, The mean, median, and mode numbers of children per family respectively are: A. 3,4.2,3 B. 3,3.4,2 C. 3,3,2 D. 2,3.5,3 E. None of the above 23. -------------- not available-------------- 24. -------------- not available-------------- 25. -------------- not available-------------- 26. The type of statistical test that can best be employed in comparison of systolic blood pressure in independent sampling of pregnant and non pregnant women.. A. Chi-square analysis B. Students t-test C. Analysis variance D. Paired t-test E. Linear regression: Q. 27-29.: Patients who have had proven myocardial infarctions MI are compared with matched controls without a history of infarction as the basis of electrocardiograms. The table below shows the 27. The sensitivity is : A. (92) B. (84) C. (16) D. (73) E. (95) 28. The false positives proportion is : A. (92) B. (84) C. (16) D. (73) E. (95) 29. The positive predictive value is A. (92) B. (84) C. (16) D. (73) E. (95) 30. The best available source of information to estimate the incidence of the meningococcal meningitis is : A. Death certificate. B. Household surveys. C. Cancer registry. D. Directorate of public health department records. E. Life insurance companies. 31. Health information system uses information related to: A. Efficiency of health service B. Effectiveness of preventive programs C. Extent of coverage of health programs D. Health personnel shortages E. All of the above 32. The most important statement about health information is: A. It must affect decision making B. It must be precise C. It must be computerized D. It must be comprehensive E. It must be easily retrieved Results Abnormal ECG Normal ECG Total MI 1100 100 1200 Healthy 400 2100 2500 Total 1500 2200 3700 A + B A B
  • 28. 2014 Exam Paper (70%)- MCQs (Duration 150 minutes) Dr.AbdulQawi Almohamadi Page 28 of 107 33. Technology has increased the cost of health care because: A. It has decreased the number of health workers B. It has provided better diagnosis C. It has improved information cost D. It often provides care but not total cure E. It often leads to better paid administrators 34. Which of the following mechanism could discourage over utilization of health services A. Full health care coverage insurance plans B. Deductibles and coinsurance C. Government supported health care delivery system D. Home health care plans E. Hospice care 35. If the prevalence of protein Calorie ( PC) under nutrition with overt signs and symptoms in a country is 2% , then: A. (PC) under nutrition is not public health problem in that country B. Subclinical (PC) under nutrition is low. C. The incidence of (PC) under nutrition is high D. The case fatality is high E. The prevalence of (PC) under nutrition is low compared with other countries 36. To conduct a comprehensive nutritional assessment, the epidemiologist usually start with: (the starting data source): A. Community cross-sectional survey B. Routine health data C. Nutritional surveillance D. Case-control study E. Ecological study 37. Air pollution: A. Has an effect only in the immediate neighborhood of the source. B. Is rapidly cleared by temperature inversions. C. Is associated with increased mortality from all causes and not just respiratory diseases. D. Humidity and mist has no synergistic effect on air pollution E. Is more important in the causation of lung cancer than tobacco smoking. 38. An important indicator of air pollution is : A. Chlorine B. Fluorine C. Sulphur dioxide D. Carbon dioxide E. Hydrogen 39. Which one of the following pesticides is capable of causing pulmonary fibrosis? A. Organophosphates B. Parathion C. Halogenated hydrocarbons D. Paraquat E. Organochlorines 40. The Dose of Gama-Ray Radiation received from a fixed source is: A. Inversely proportional to the square of distance B. Directly proportional to the distance C. Directly proportional to the square of the distance D. Expressed as 3 times the product of the distance and time. E. Expressed as the sum of the squares of the distance and rate of exposure. 41. The best method for treating water from a very turbid source is: A. Aluminum coagulation B. Rapid sand filtration C. Activated charcoal D. Ozonation E. Aeration 42. Testing For Albuminoidal Nitrogen And Nitrites Would Best Be Utilized In: A. Air pollution control B. Sewage processing C. Pasteurization of milk D. Water purification E. Food processing 43. Sulfate of Aluminum is commonly utilized in A. Milk purification B. Decreasing air pollution C. Prevention of radiation hazard D. Sewage disposal E. Water purification 44. The object of disinfection of water is to: A. Reduce the bacterial population B. Destroy bacterial population C. Prevent bacterial toxin formation D. Limit growth of pathogenic bacteria E. None of the above 45. Which of following has been LEAST implicated in diseases spread by milk and milk products? A. Cream B. Certified milk C. Butter D. Butter milk E. Ice cream 46. DDT is an effective insecticide that operates as a: A. Repellant B. CNS poison C. Bactericide D. Stomach poison E. Fertility inhibitor
  • 29. 2014 Exam Paper (70%)- MCQs (Duration 150 minutes) Dr.AbdulQawi Almohamadi Page 29 of 107 47. Which of the following is the most resistant to water disinfection. A. Virus of infections hepatitis B. Schistosomiasis C. Coliform organisms D. Cysts of E. histolytica E. Enteric bacteria 48. An electrostatic precipitator is best used in: A. Air pollution control B. Sewage disposal C. Pasteurization of milk D. Water purification E. Arthropod-vector control 49. The cheapest method of refuse disposal is: A. Incineration B. Dumping at sea C. Reduction Sanitary D. Sanitary fill E. Feeding to animals 50. The Phosphatase test is associated with: A. Water purification B. Pasteurization of milk C. Sewage disposal D. Air pollution control E. None of the above 51. Chemical food poisoning associated with preparations added to raw meat to preserve the red color, is most commonly due to : A. Antimony B. Arsenic C. Cadmium D. Nicotinic acid E. Nitrates 52. Which of the following general effects would be most character of an ascent to an altitude of 12.000 feet? A. Deterioration of voluntary muscle control B. Impairment of ocular balance C. Impairment of hearing D. Oxygen would be required at night E. Visual acuity less than 50% of normal 53. Which of the following does not apply to carbon monoxide? A. Always present in the exhaust from internal combustion engines B. Odorless C. Tasteless D. Has a lesser affinity for hemoglobin than oxygen E. Non-irritating 54. Which of the following dusts predispose to tuberculosis? A. Asbestos B. Zinc C. Silica D. Graphite E. None of the above 55. Major purpose of medical service in industry: A. To prevent job related injury or illness B. To provide emergency medical aid C. Effective employee placement D. To maintain good health for the total personnel by health promotion and health education. E. All of the above 56. A patient with headache, nausea, vomiting, fatigue, jaundice, hepatomegaly and oliguria, may have an occupational disease caused by: A. Lead B. Mercury C. Benzol D. Carbon tetrachloride E. Hydrogen sulfide 57. Damage to reproductive cells may occur as a result of occupational exposure to A. Chromium. B. Lead C. Mercury D. Zinc E. Carbon monoxide 58. Caisson disease is the result of: A. Swelling of synovial tissues B. Carbon dioxide saturation in the blood C. Air in the abdomen D. Nitrogen bubbles in the blood stream E. Oxygen bubbles in soft tissues. 59. Which of the following effects has not been demonstrated to be the result of industrial noise? A. Increased peristalsis B. Increased blood pressure C. Decreased gastric secretion D. Irregular heart action E. Fatigue 60. Which of the following causes the highest loss of work days: A. Circulatory diseases B. Bone and joint diseases C. Digestive disorders D. Allergic disorders E. Nutritional diseases
  • 30. 2014 Exam Paper (70%)- MCQs (Duration 150 minutes) Dr.AbdulQawi Almohamadi Page 30 of 107 61. Persons at an increased risk of bladder cancer include workers in all the following industries EXCEPT: A. Manufacture of rubber B. Dyeing of textiles C. Manufacture of paints D. Manufacture of steel E. Printing 62. Which of the following substances is causally associated with pneumoconiosis? A. Sulfur oxides B. Nitrogen oxides C. Oil fumes D. Dust particles E. Cigarette smoke 63. The major environmental source of lead absorbed in the human blood stream is: A. Air B. Water C. Lead-based paint D. Food E. None of the above 64. Prolonged exposure to polyvinyl chlorides in production is association with each of the following EXCEPT: A. Acroosteolysis B. Raynaud disease C. Lung disease D. Angiosarcoma of the liver E. Scleroderma 65. The most common asbestos-related tumor in human is A. Bronchogenic carcinoma B. Carcinoma of the colon C. Pleural mesothelioma D. Peritoneal mesothelioma E. Pericardial mesothelioma 66. Ozone depletion due to air pollution is controlled by: A. Using lead free Fuel! B. Reduction of world production of Freon C. Filtration of exhaust D. Wide use of electric energy instead of Fuels E. Banning the use of refrigerators. 67. The development of neuropathic symptoms is associated with chronic exposure to all the following substance EXCEPT: A. Mercury B. Lead C. Arsenic D. Sulfur dioxide E. Nitrous oxide 68. The most prevalent mental disorder in young children is A. Autism B. Mental retardation C. Behavioral problem 69. ---------- Unavailable ……………. 70. ---------- Unavailable ……………. 71. ---------- Unavailable ……………. 72. The most prevalent psychiatric disorder among opiate addicts undergoing treatment is A. Schizophrenic B. Depression C. Alcoholism D. Antisocial E. Mania 73. In DOTS all the following are true EXCEPT: A. Can solve the problem of defaults B. Is a program with a high cost effectiveness C. Is designed to be used by trained doctors D. Can prevent the development of drug resistance E. Has a up to 96% cure rate 74. B.C.G. vaccination: A. Is given to all tuberculin positive cases in endemic areas B. Is given by intramuscular injection at deltoid region C. Is not recommended to be given to newly born babies D. Can give a partial or complete immunity for up to 12 years E. Can cause superlative lymphadenitis in 10% of the cases 75. Brucella species can: A. Survive direct sunlight for four days B. Stand boiling for few hours C. Stand freezing for several weeks D. Be transmitted by yogurt made from sheep milk E. Easily transmitted from a patient to his direct contacts 76. In the transmission of cholera A. Direct spread among contacts is very important during epidemics B. Is rare in Europe because of cold climate C. The highest rate is usually among young adults during epidemics D. Asymptomatic cases are commoner among classical biotype E. Long term carries are common
  • 31. 2014 Exam Paper (70%)- MCQs (Duration 150 minutes) Dr.AbdulQawi Almohamadi Page 31 of 107 77. A 11 years people was diagnosis to have Meningococcal meningitis, the most important control measure in the school is A. Mass vaccination B. Chemoprophylaxis by rifampicin C. Close surveillance of contacts. D. Good ventilation of classes E. Improve the sanitation of the school. 78. All the following are true in typhoid fever EXCEPT: A. In every outbreak there should be a human carrier. B. Fecal carries are usually associated with gall bladder disease. C. Vaccination by TAB (typhoid and paratyphoid A&B) is superior to that by Ty21a. D. A negative Widal test might be more useful in exclusion the disease. E. The usual incubation period is 10-14 days. 79. In Crimean- cango hemorrhagic fever? A. The disease is limited to the Africa and Crimean. B. The mortality rate is 2-5%. C. The reservoir is among ticks. D. Transmission by mosquitoes E. There are no subclinical cases. 80. In Rift Valley Fever all the following are true EXCEPT: A. Mammals are the reservoir. B. Has a low mortality rate among the human. C. The vector is mites. D. Has recently cross the red sea to the Yemen and Saudi Arabia. E. Can cause hemorrhagic manifestation. 81. The aims for Control Diarrhea Disease (CDD) program are the following EXCEPT: A. Epidemiologic surveillance for epidemics of the disease B. Initiating an ORT room is all PHC centers C. Training mothers for proper and scientific use of antibiotics D. Promotion of breast feeding E. Maintaining acute statistics 82. The following can increase the possibility of sexual transmission of HIV infection EXCEPT: A. Accompanied ulcerative STD B. Very early infection C. Late presentation of AIDS D. Circumcision E. Rectal sex 83. Chlamydia species can cause the following diseases in man EXCEPT : A. Non-specific cervicitis B. Trachoma C. Lymph granuloma veneered D. Chancroid E. Psittacosis 84. An influenza pandemic is usually expected when: A. There is an antigenic shift in type B B. There is an antigenic drift in type B C. The isolated strain is Hong Kong type C D. There is an antigenic shift in type A E. There is a direct transmission from swine to man 85. Amoebic dysentery is differentiated from bacillary dysentery by having more tendency to be: A. Transmitted directly from man to man B. Associated with prostration C. Associated with tenesmus D. Presented as sporadic cases E. Easily transmitted by water 86. The following are true association in cancer EXCEPT: A. Cigarette smoking and bladder cancer B. Sexual promiscuity and cervical cancer C. Vinyl chloride and liver angiosarcoma D. Early menopause and breast cancer E. Nullipara and ovarian cancer 87. In viral hepatitis type C: A. Screening test is not yet available B. Vertical transmission is common C. Immunization is readily available D. The incubation period is 5-7 weeks E. Animal reservoir is important 88. Transplacental transmission can occur in all the following disease EXCEPT: A. Rubella B. Cytomegalovirus infection C. Toxoplasmosis D. Hepatitis A infection E. Syphilis 89. The first step in conducting an epidemic investigation is to: A. Determine the number of cases B. Calculate the incubation period C. Determine the population at risk D. Verify the diagnosis E. Collect appropriate samples
  • 32. 2014 Exam Paper (70%)- MCQs (Duration 150 minutes) Dr.AbdulQawi Almohamadi Page 32 of 107 90. The best measure of obstetric care quality in a community is: A. Crude death rate B. Infant mortality rate C. Abortion rate D. Perinatal mortality rate E. None of the above 91. Which of the following is most characteristic of the abuser of the elderly? A. Encourages of the elderly person to speak his or her mind B. Has cared for the elderly person for some time C. Lives separately from the abused person D. Takes immaculate care of the house and the abused person E. Volunteered to take care of the abused person 92. The longitudinal research approach is characterized by: A. The same group being studied over extended periods of time B. Different groups being studied over extended periods of time C. Individual being studied at different intervals throughout their life cycle D. Groups of individual of overlapping ages being tested periodically in similar dimensions E. Different dimensions of behavior being examined at different stages of development to determine short-range effects of longitudinal analysis 93. Death from overdose is a strong possibly with all of the following drugs EXCEPT: A. Alcohol B. Narcotics C. Sedative-hypnotics D. Psychedelics (including marijuana) E. Phencyclidine 94. A confounding variable may relate closely in time in the of disease but: A. Is not necessary for its occurrence B. Is one of several causal agents C. Is more difficult to measure than other variables D. Exacerbates rather than causes the disease E. Can't be alleviated through disease prevention programs. 95. Which of the following is characteristic of a single exposure, common vehicle outbreak? A. Frequent secondary cases B. Increasing severity with increasing age C. Explosive D. Cases include in both people who have been exposed and those didn't exposed. E. All of the above 96. In the study of an outbreak of an infectious disease plotting an epidemic curve is useful because A. It helps determine what type of outbreak (e.g. single-space, person-to-person) has occurred B. It shows whether herd Immunity has occurred C. It helps determine the median incubation period D. a and c E. a,b and c 97. Which of the following is/are a good index of the severity of short-term acute disease: A. Cause-specific death rate B. 5-year survival C. Case-fatality rate D. Standardized mortality ratio E. None of the above 98. Ecologic fallacy refers to: A. Assessing exposure in large groups rather than in many small groups B. Assessing outcome in large groups rather than in many small groups C. Ascribing the characteristics of a group to every individual in that group D. Examining correlations of exposure and outcomes rather than time trends E. Failure to examine temporal relations between exposures and outcomes 99. Diabetes mellitus A. Is easily defined for clinical and epidemiological purposes B. Prevalence increases with age C. Dose not affect life expectancy D. Usually causes death through its specific complications E. Occurs only in industrialized, sedentary, Western populations 100. The risk of coronary heart disease from smoking cigarettes : A. Shows the greatest relative risk in the younger age groups B. Appears to be irreversible in those who stop smoking C. Accounts for less smoking related deaths than does lung cancer D. Has a greater relative risk than does smoking for lung cancer E. Operates in populations with very low serum cholesterol levels. 101. The following are Modifiable Risk Factors of Hypertension EXCEPT: A. Obesity B. Genetic Factors C. Stress D. Excess salt intake E. Oral contraceptive use